Está en la página 1de 48

w

U
o
z
,--

01 . Semiologa urolgica 03. Urolitiasis 09


y definiciones 01
3.l. Epidemiologia 09
3.2. Manifestaciones clnicas
1.l. Definiciones 01
1.2.
y su manejo agudo 10
Diagnstico diferencial
3.3. Evaluacin y tratamiento
de la hematuria macroscpica 02
de la litiasis renal 11

02. Infecciones
04. Tumores renales 15
del tracto urinario.
Cistitis intersticial 03 4.1. Carcinoma de clulas renales
(adenocarcinoma renal, hipernefroma) 15
2.l. Patognesis y etiologa 03 4.2. Otros tumores 17
2.2. Diagnstico 04
2.3. Diferentes ITU y su tratamiento 04
2.4. Tuberculosis genitourinaria 07 05. Hiperplasia
2.5. Cistitis intersticial 07
y carcinoma prosttico 19

S.l. Hiperplasia prosttica benigna 19


5.2. Carcinoma prosttico 21

VI
06. Carcinomas 09. Uropata obstructiva 38
del tracto urinario 26
9.l. Caractersticas 38
9.2. Clnica 38
6.l. Carcinoma vesical 26
9.3. Diagnstico 39
6.2. Tumores del tracto urinario superior 28
9.4. Tratamiento 39

07. Tumores testiculares 31


1 0. Disfuncin erctil 40
7.1. Etiologa y epidemiologa 31
10.1. Introduccin 40
7.2. Anatoma patolgica 32
10.2. Prevalencia 40
7.3. Clnica 32
10.3. Etiologa 40
7.4. Diagnstico 32
10.4. Factores de riesgo 40
7.5- Diagnstico diferencial 33
10.5- Diagnstico 41
7.6. Tratamiento 34
10.6. Tratamiento 41

08. Trasplante renal 36


Bibl iografa 42
8.l. Indicaciones 36
8.2. Contraindicaciones 36
8.3. Complicaciones 37

VII
01

SEMIOLOGA UROLGICA
y DEFINICIONES

Orientacin Aspectos esenciales

MIR
Este tema no se ha preguntado
en el MIR de forma directa.
GJ La causa ms frecuente de hematuria microscpica es la litiasis (poblacin general, ambos sexos).

Puede ayudar a obtener una la causa ms comn de hematuria microscpica en varones de ms de 50 aos es la hiperplasia benigna
visin general de la materia
III
de prstata.
ya asociar algunos hallazgos
a patologras concretas, pero la hematuria con cogulos indica un problema urolgico.
no se debe emplear en l
demasiado tiempo. la causa ms habitual de hematuria es la cistitis hemorrgica, pero 10 primero a descartar es el tumor uro
leJia!.

IIJ Los hemates dismrficos en el sedimento orientan a nefropala de origen glomerular.

1 . 1 . Defin iciones

Hematuria microscpica: presencia de ms de 5 hemates por campo. La causa ms frecuente en ambos sexos
es la litiasis. La causa mas comn en varones mayores de 50 aos es la hiperplasia benigna de prstata.
Hematuria macroscpica: orina de aspecto rojizo a simple vista debido a la presencia de ms de 50 hemates
por campo. En los pacientes fumadores, en ausencia de otros sntomas, se debe sospechar tumor urotelial.
Piuria: presencia de ms de 10 leucocitos por campo. Altamente nespecfica, pero en presencia de sntomas
urinarios, hay que sospechar infeccin.
Sndrome miccional: presencia de polaquiuria (aumento en la frecuencia miccional), urgencia miccional (ne
cesidad imperiosa e irrefrenable de orinar) y disuria (molestias urinarias inespecficas referidas como ardor,
escozor, etc.).
Incontinencia urinaria: prdidas involuntarias de orina (Tabla 1 l. Existen cuatro tipos principales:

INCONTINENCIA INCONTINENCIA
SINTOMAS
DE URGENCIA DE ESFUERZO

Urgencia (deseo rwpentIno de ortnar) Si No

Aumento de laheuencla mlcdonal Si No

Copoc:ldod de __ .. bollo despus de _ 01 deseo de orinar No SI

_Ir"boIio_la_ sr Generalmente no

Escape _la lICtMdad fIsIca No Si

eanddod de orIno ....poda


. en coda opIlIOdlo de IncontI"'''cla Abundante. si se produce Generalmente escasa

Tabla 1. Diagnstico diferencial de la cl nica de incontinencia urinaria femenina

Continua: de da y de noche, en todas las posiciones. La causa ms frecuente es la fstula urinaria (en pacientes
con antecedentes quirrgicos previos) y la segunda, el urter ectpico (que es la causa ms frecuente en nias).
De esfuerzo: se desencadena con el aumento de presin abdominal (al rer, toser, cargar con peso). Ge
neralmente se produce por un dficit de soporte de la musculatura perineal (por ello es recomendable
? Preguntas
revisar los antecedentes obsttricos, pacientes obesas, pacientes aosas, ete.).

- No hay pregunta s MIR


De urgencia: el paciente siente ganas de orinar, pero no le da tiempo a l legar al bao (provocadas por
representativas. contracciones involuntarias del msculo detrusor).
Manual CTO de Medicina y Cirugfa, 8.a edicin

Mixta: generalmente es una combinacin de las dos anteriores.


1 .2. Diagnstico d i ferencia l
Paradjica: escape de orina debido a la sobreclistensin vesical.
El ejemplo caracterstico es el paciente prosttico con retencin d e l a hematuria macroscpica
urinaria. La presin i ntravesical supera la presin de cierre del
esfnter uretral, producindose un escape de orina paradjico
(no puede orinar y, sin embargo, se le escapa la orina). Segn el momento de aparicin:
Inicial: sangrado uretral o prosttico.

RECUERDA Final: sangrado del cuello vesical.


Total: vesical o del tracto urinario alto. Se debe recordar que un
los cilindros hemticos aparecen en las glomerulonefritis que producen
sndrome nefrtico, como en la postinfecciosa. sangrado importante de cualquier parte del aparato genitourina
rio puede provocar hematuria total.
Hematuria con cogulos: indica un problema urolgico. La cau
Enuresis: prdidas de orina exclusivamente durante el sueo. Si el sa ms frecuente en mujeres es la cistitis hemorrgica, aunque la
nio es mayor de 6 aos, debe ser estudiado. primera causa a descartar es una neoplasia urotelial, mxime en el
Crisis renoureteral: dolor lumbar frecuentemente irradiado a geni paciente fumador.
tales, de carcter agudo, cuya intensidad no se modifica por los Hematuria por nefropata mdica: no suele tener cogulos, y puede
cambios pastura les, y que se suele acompaar de nuseas, vmitos ir acompaada de cierto grado de proteinuria, as como de cilindros
y malestar general. Es muy poco frecuente que sea bilateral. eritrocitarios o de hemates dismrficos en el sedimento urinario.

2
02.
INFECCIONES DEL TRACTO URINARIO.
CISTITIS INTERSTICIAL

Orientacin

MIR
la causa ms frec uente de infeccin del tracto urinario OTU) es Escherichia coli, tanto a nivel comunitario
Este tema es el ms importante
como nosocomial.
de esta asignatura. Se debe
conocer muy bien, pues dos El origen ms frecuente de uretritis es Chlamydia lrachomalis.
o tres preguntas son habituales
en el examen. El estudio de las la causa habitual de orquiepididimitis depende de la edad: Chlamydia y gonococo si es menor de 35 aos;
preguntas de otros aos suele enterobacterias, si es mayor de esa edad.
ser de gran ayuda, ya que son
baStlnle repetitivas. No se la causa ms frecuente de absceso renal en el UDVP es Staphylococcus aureus.
debe bajar la guardia con l a
tuberculosis genitaurinaria n i El diagnstico definitivo de ITU es microbiolgico: ms de '05 UFClm1. No obstante, este criterio vara con
e n la cistitis intersticial. Hay el sistema de recogida.
que formar una imagen mental
tpica para reconocerlas en Si se recoge la muestra urinaria mediante puncin suprapbica, cualquier nmero de bacterias es significativo.
caso clnico, que es como
la bacteriuria asintomtica se trata en gestantes, menores de 5 aos, inmunodeprimidos, previamente a la
suelen preguntarlas.
o ciruga urolgica, o si la especie implicada es Proteus.

Los sistemas de drenaje cerrados son preferibles a los abiertos, pues la lasa de infeccin es menor.

La infeccin del tracto urinario (lTU) puede clasificarse de varias formas. Se puede hacer una divisin anatmica
entre las ITU altas (infecciones renales) y las ITU bajas (cistouretritis, prostatitis). Asimismo, la clasificacin pue
de basarse en la asociacin o no de complicaciones. Una ITU no complicada es un cuadro clnico caracterizado
por la presencia de escozor miccional, urgencia y frecuencia, acompaado o no por hematuria terminal, dolor
hipogstrico, y ms raramente, febrcula. Dentro de este grupo se podran incluir las pielonefritis no complica
das, que se presentan como cuadros febriles con hipersensibilidad en fosa lumbar, fiebre, nuseas o vmitos, y
sin los factores que convierten la [TU en "complicada", como son: presencia de catteres, uropata obstructiva,
reflujo vesicoureteral, anomalas anatmicas, insuficiencia renal o trasplante renal. La ITU en el varn debe
considerarse esencialmente "complicada" de entrada.

La reaparicin de una infeccin tras el tratamiento puede deberse a reinfeccin o recidiva. El primer trmino expre
sa la infeccin nueva por un germen distinto al inicial, mientras que recidiva indica infeccin por el mismo germen.
Esta ltima es mucho ms infrecuente que la reinfeccin y puede estar ocasionada por litiasis infectiva, prostatitis
crnica, fstulas vaginales o intestinales, divertculos vesicales infectados, cuerpos extraos, necrosis papilar infec
tada y otras causas que generan un reservario de microorganismos que difcilmente se eliminan con el antibitico.

2 . 1 . Patognesis y etiolog a

Existen tres posibles vas por las que los microorganismos pueden alcanzar el tracto urinario: hematgena,
III Preguntas linftica y ascendente. La va l inftica carece de i mportancia real. La diseminacin hematgena tampoco es
ferecuente. La ms comn es la ascendente i niciada en la uretra. Probablemente por esta razn es mucho ms
- MIR 08-09,98
habitual la ITU en mujeres, dado que su uretra es muy corta y ancha, y por ello favorece el paso de microorga
- MIR 05-06, 230
-MIR 03-04, 78,8 4 nismos hacia niveles ms altos del TGU.
- MIR 02-03, 1 34,1 74
-MIR OJ-02 , 103, 1 06
-MIR OO-01F,1 34, 1 45 Otro dato que apoya la importancia de la va ascendente es la frecuencia de i nfeccin tras el cateterismo
- MIR 99-00, 1 35 uretral, que es del 1 % en los pacientes ambulantes, y en tres o cuatro das alcanza a casi la totalidad de los
- MIR 99-00f,1 1 9
- MIR 98-99F, 1 1 8
pacientes sondados con sistemas de drenaje abiertos. En los enfermos hospitalizados, el riesgo de i nfeccin
- MIR 97-98, 26,206, 2 1 5 alcanza un 5% por cada da de sondaje, incluso con sistemas cerrados (MIR 0 1 -02, 103).

3
Manual CTO de Medicina y Ciruga, 8.a edicin

Una vez que las bacterias han alcanzado el tracto urinario, tres factores 2.3. Diferentes ITU y su trata m ie nto
determinan el desarrollo de la infeccin:
La virulencia del microorganismo.
El tamao del inculo.
Los mecanismos de defensa del husped. En el tratamiento de la ITU lgicamente es fundamental el empleo de
antimicrobianos. El nmero de stos empleado es elevado y las pautas
La mayora de las infecciones en la comunidad estn producidas por de tratamiento muy variables. A continuacin, se repasarn las opcio
grmenes gramnegativos, principalmente E. coli (MIR 05-06, 230), res nes teraputicas segn el tipo de ITU a la que uno se enfrente.
ponsable del 85% y, en menor proporcin, Proteus, Klebsie/la o Pseu
domonas (MIR 97-98, 2 1 5). Entre los grampositivos, nicamente el Sta
phylococcus saprophyticus tiene relevancia, produciendo el 10- 1 5% Bacteriuria asi ntomtica
de las ITU en mujeres jvenes (segundo germen ms frecuente en esta
poblacin).
Definida como bacteriuria significativa (105 UFC/ml) en a l menos dos
Alrededor del 30% de las mujeres con clnica miccional presentan re urocultivos con el mismo germen, tomados con una semana de dife
cuentos menores de 105 colonias por mililitro ( 1 05 UFC/ml); de stas, rencia en ausencia de sntomas. la bacteriuria asintomtica no debe
tres cuartas partes presentan piuria; en el resto, existen pocos datos tratarse salvo en los casos en los que conlleva un riesgo de i nfeccin
que demuestren infeccin, y en general se tratan segn la clnica. En clnica o dao orgnico, como ocurre en nios menores de 5 aos,
la orina de las pacientes sintomticas con piuria, se pueden encontrar tengan o no patologa urolgica asociada. Asimismo, debe ser tratada
(considerndose infeccin activa) recuentos ms bajos (1 02- 1 04) de en el embarazo (MIR 08-09, 98 ; M I R 03-04, 78), en pacientes inmu
los patgenos habituales. En otras ocasiones, el cuadro se justifica por nodeprimidos, como profilaxis previa a una ciruga urolgica y en los
la presencia de uretritis causada por N. gonorrhoeae o C. trachomatis. casos de bacteriuria por Proteus (MIR 02-03, 1 34) (Tabla 2).
El papel patgeno de grmenes como U. urealyticum o Mycoplasma
hominis est mal definido, ya que se desconoce su potencial como
Embarazadas
uropatgenos aislados (MIR 99-00F, 119).
Inmunodeprimidos
Previamente a ciruga urolgica
En las i nfecciones nosocomiales, los grmenes gramnegativos conti
Bacteriuria por Proteus
nan siendo los ms frecuentes. Si bien E. coli es el ms habitual, su
frecuencia desciende hasta el 50% y adquieren mayor importancia Tabla 2. Bacterruria asintomtica: indicaciones de tratamiento
Proteus, Klebsiella, Pseudomonas, Enlerobacler y Serratia (MIR 03-04,
84). El 25% restante est ocasionado por grmenes grampositivos como
estreptococos y estafilococos. Candida albicans puede aparecer prin
cipalmente en pacientes diabticos, cateterizados o con tratamientos
D RECUERDA
Proteus es intrnsecamente resistente ti las nitrofurantonas, ya que al
antibiticos prolongados. caliniza la orina gracias a su ureasa, y este grupo de antibiticos nica
mente es til en medio cido.

la afectacin del tracto urinario superior parece tambin producirse


por ascenso de los grmenes a lo largo del urter. La diferenciacin,
aunque poco especfica, se debe basar en los hallazgos clnicos (fiebre, En el caso de los pacientes sondados permanentemente, la presencia
dolor lumbar, escalofros) y analtica elemental ( leucocitosis, velocidad de bacteriuria asintomtica no es una indicacin de tratamiento y,
de sedimentacin alta). actualmente, incluso es dudosa la recomendacin clsica de empleo
profilctico de algn antibitico, previo a la sustitucin del catter, a
fin de contrarrestar la posible diseminacin hematgena del germen
producida por la manipulacin (las ltimas guas clnicas ya no lo reco
2.2. D i a g nstico miendan). S es, sin embargo, i ndicacin de tratamiento la bacteriuria
persistente a los 3-5 das de haber retirado una sonda vesical. En aque
llos pacientes en los que la sonda no pueda ser retirada, el tratamien
E l d i agnstico de ITU, adems de la clnica, se define por el cultivo to de las bacteriurias asi ntomticas no suele ser efectivo, y puede dar
de orina. Dado que es frecuente el crecimiento de bacterias que han lugar a seleccin de cepas resistentes. En estos pacientes slo se debe
contaminado las muestras, se utiliza u n criterio estadstico sobre la iniciar tratamiento si presentan alto riesgo de desarrollar bacteriemia o
base del recuento de colonias del urocultivo, considerando como si la bacteriuria se hace sintomtica.
significativo clsicamente el crecimi ento de ms de 1 05 colonias por
m i l i l itro (MIR 97-98, 206). En determinadas circunstancias, recuen En el resto de los casos, nicamente con la concurrencia de factores
tos de colonias menores pueden ser suficientes: recuentos de 1 03 particulares, se debe tratar la bacteriuria, y siempre sobre la base del
UFC/ml en mujeres sintomticas, ms de 1 04 en pielonefritis c l nicas estudio de sensibil idades (MIR 98-99F, 1 1 8).
o en varones, y ms de 1 02 en muestras de cateterismos l i mpios
o cualquier recuento, si se recoge mediante puncin-aspiracin su
prapbica (MIR 97-98, 26). Cifras mayores de 105 U FC/ml pueden ITU baja en mujeres
igualmente reflejar contaminacin, principalmente si crecen dos o
ms especies.
Puede realizarse un tratamiento convencional de siete das o bien u n
En el adulto, la presencia de piuria (ms de 1 0 leucocitos/mm3) se rela curso corto e n monodosis o en rgimen d e tres das. L a ventaja d e stos
ciona estrechamente con la ITU en presencia de sntomas, no as en el es el menor coste econmico y la menor incidencia de efectos adver
nio, en el que puede acompaar a los cuadros febriles. sos. Su desventaja es la mayor incidencia de recurrencias tempranas, a l

4
urologa a
no afectar apenas a los reservorios vaginal e intestinal de uropatgenos. El antibitico empleado se seleccionar, por supuesto, sobre l a
Aun con todo, por las ventajas mencionadas, la pauta preferida actual base del cultivo y d e l antibiograma, y cuando s e i n icie d e forma
mente es el tratamiento de tres das. emprica, habr que tener en cuenta factores que orienten hacia
el germen causante: mayor incidencia de Pseudomonas en perso
Los antibiticos de eleccin son el cotrimoxazol, las fluoroquinolonas nas diabticas y enfermos de UVI, estafilococo en adictos a drogas
y la amoxicilina-cido clavulnico, fosfomicina o n itrofurantona. parenterales (MIR 03-04, 84), Proteus en pacientes con litiasis in
fectiva, presencia de sondas, catteres, tratamientos antibiticos
En mujeres embarazadas se recomiendan las pautas largas de trata previos, etc.
miento (siete das), evitando el uso de sulfamidas al final del embarazo
por el riesgo incrementado de kernicterus, y el empleo de quinolonas En la evaluacin del paciente con pielonefritis y mala respuesta a tra
por el dao producido sobre el cartlago de crecimiento fetal. Tampoco tamiento inicial, es recomendable la realizacin de una ecografa para
se emplearn pautas cortas en caso de sospecha de pielonefritis, pre descartar obstruccin o litiasis (M I R 00-01 F, 134).
sencia de clculos o anomalas de la va urinaria, o bien infecciones
previas por microorganismos resistentes a los antibiticos. ITU en varones

ITU recurrente Cualquier ITU en varn debe considerarse como complicada inicial
mente ya que hay que asumir que existe afectacin del tejido prost
tico, renal o que existen problemas concomitantes como obstruccin
Aparicin de cuatro o ms episodios al ao. Se puede realizar profi urinaria, litiasis o malformaciones urolgicas. Por todo ello, el trata
laxis con cotrimoxazol O una fluoroquinolona (en funcin de la sensi miento debe ser ms prolongado (mnimo una semana), no siendo ade
bilidad del germen aislado en el ltimo episodio) en dosis nica, das cuados los cursos cortos de tratamiento.
alternos, durante seis meses. Si tras la retirada se presentaran nuevas
recurrencias, puede reinstaurarse el tratamiento durante periodos ms
prolongados (1-2 aos). Es aconsejable la ingesta abundante de agua Prostatitis
y realizar micciones frecuentes y cumplir una serie de reglas bsicas
higinico-dietticas.
La infeccin aguda del tej ido prosttico se presenta como un cuadro
Si los episodios tienen relacin con el coito, se puede administrar un sptico con afectacin general del paciente, fiebre elevada, sndrome
comprimido de cotrimoxazol o una quinolona despus del mismo. En miccional, artromialgias y dificultad miccional (Tabla 3). En el examen
mujeres posmenopusicas, el tratamiento con estrgenos tpicos vagi rectal, la prstata aparece muy dolorosa e inflamada. El germen ms
nales disminuye la frecuencia de i nfecciones. habitual es E. coli. Durante la inflamacin aguda, los antibiticos pe
netran adecuadamente, pero una vez que sta cede, la penetracin es

RECUERDA ms pobre. Por ello, se deben utilizar cursos largos de tratamiento (3-4
semanas) para i n tentar evitar la persistencia de focos que den pie a una
Staphylococcus saprophylicus se ha relacionado con ITU en mujeres
jvenes sexualmente activas. prostatitis crnica. Entre los antimicrobianos empleados, las fluoroqui
nolonas son las que mejor difunden al tejido prosttico .

Pielone!ritis aguda no complicada


RECUERDA
En pacientes con SIDA, Cryptococcus neoformans puede ser una causa
de prostatitis, ya que se elimina a travs de la orina.
En los casos de gravedad leve-moderada, puede plantearse terapu
tica oral con cotrimoxazol (en desuso en nuestro medio por el ele
vado ndice de resistencias), fluoroquinolonas o B- Iactmicos. En La prostatitis crnica bacteriana suele presentarse como molestias
pacientes graves u hospita l i zados es preciso tratamiento parenteral, perineales o genitales, sntomas i rritativos (polaquiuria, tenesmo, es
y el espectro de antimicrobianos incluye ampicilina (enterococo), cozor) y episodios de ITU recurrentes causados por el mismo organis
ureidopen icilinas (Pseudomonas), cefalosporinas de segunda o ter mo. En el lquido prosttico se evidencian ms de 10 leucocitos por
cera generacin, e incluso aminoglucsidos. Nunca se emplearn campo de gran aumento, y macrfagos que contienen cuerpos ovales
pautas cortas. grasos.

CULTIVO LIQUIDO
ETIOLOGIA ClINICA H 'ITU CULTIVO ORINA LIQUIDO PROSTATlCO TRATAMIENTO
PROSTATICO

Cotrimoxazol,
-
E.coH Cuadro sptico + + Nunca hacer masaje prosttico ni sondaje fluoroquinolonas
ogudo
4 semanas
Prostadtls Irritativo con Cotrimoxazol,
crnica E.coJ/ reagudizaciones, sin + +/- > 10 leucocitos/campo + fluoroqulnolonas
- fiebre ni leucocitosis 6-12 semanas
-
Ureaplasma Cronicidad,
crnica - - > 10 leucocitos/campo - Doxicidina
Myeoplasma empeoramiento
no*-no
a-bloqueantes
- Desconocida Oscilante - -
< 10 leucocitos/campo -
Relajantes musculares

Tabla 3. Oiagnstko diferencial de las prostatitis

5
Manual CTO de Medicina y Ciruga, B." edicin

El tratamiento debe estar guiado por los cultivos, tanto de orina como
de fluido obtenido por masaje prosttico, y prolongarse entre 4 y 1 6
semanas. Cuando se encuentra a un paciente con datos de prostatitis
crnica y signos inflamatorios en el lquido prosttico, pero sin historia
documentada de ITU y con cultivos negativos, el cuadro se denomina
prostatitis no bacteriana. En ocasiones, el responsable puede ser U.
urealylicum o M . hominis, pudiendo ser tratados estos casos con doxi
ciclina o eritromicina, sobre esta sospecha.

Se denomina prostatodinia a un cuadro clnico similar donde predo


minan las molestias perineales o genitales con cultivos negativos y
menos de 1 0 leucocitos por campo en el lquido prosttico. Su cau
sa es desconocida y el tratamiento difcil, emplendose actualmente
p-bloqueantes o relajantes musculares como terapia inicial.

Orqu iepididimitis

En varones adultos menores de 3 5 aos e s considerada, en el plano


terico, una enfermedad d e transmisin sexual, siendo los agentes
ms frecuentes Chlam ydia trachomatis y Neisseria gonorrhoeae (Ta
bla 4). Por encima de 35 aos, los microorgan i smos ms frecuentes
son las enterobacterias. El tratamiento puede l l evarse a cabo con las
sigui entes pautas: 1 ) quinolonas, 2) ceftriaxona en dosis nica i . m .
( 1 2 5-250 mg) ms 1 0 das de doxiciclina ( 1 00 mgl1 2 h/7 das), se
aplicar esta pauta en aquellos casos en los que se sospeche ETS
(MIR 99-00, 1 35).

Absceso renal

Los abscesos medulares o corticales suelen proceder de un foco de pie


lonefritis contiguo O de diseminacin hematgena de s. aureus/ proce
dente de focos cutneos en sujetos adictos a drogas por va parenteral .
E l urocultivo e n este ltimo caso puede ser negativo. El diagnstico
ms fiable se realiza mediante TC Deben tratarse con antibiticos El diagnstico es similar al absceso renal, y su tratamiento pasa por
por va intravenosa y, dependiendo del tamao y de la evolucin, el drenaje percutneo o quirrgico, con la adecuada cobertura anti
se hace obligatorio el drenaje mediante puncin percutnea O quirr bitica.
gicamente.

ITU asociada a catteres


Absceso pe,i"enal

Se localiza entre la cpsula renal y la fascia de Gerota. Lo ms fre La ITU es la infeccin hospitalaria ms frecuente, y los catteres urina
cuente es que un absceso cortical se abra a este espacio, pero pue rios la principal fuente de sepsis. Se calcula que el 1 % de cateterismos
d e ocurrir tambin por diseminacin hematgena. El germen ms ambulatorios transitorios sufren una ITU posterior y que la mayora de
frecuente es E. coli, y enfermos con catter permanente presentan una bacteriuria significati
D RECUERDA S. aureus en los casos va al cuarto da de su colocacin. Esta bacteriuria puede hacerse sinto
S. oureus es tambin la causa ms frecuente
de diseminacin he mtica en forma de cuadros de cistitis, hematuria o episodios febriles,
de endocarditis Infecciosa.
matgena (Figura 1). muchas veces autolimitados.

ENFERMEDAD ETIOLOGIA LESiN TIPICA DIAGNSTICO TRATAMIENTO

Neisserio gonorrhoeoe Asintomtica () Contacto < 5 dlas Ceftriaxona o


Exudacin uretral matutina (1;) Gram de exudado cervical espectinomicina (no en
Epididimoprostatitis, salpingitis, sndrome Cultivo en medio de Thayer-Martin farngeas)Ciprofloxacino
Uretritis gonoccIco
Fitz-Hugh-Curtis, gonococemia diseminada
(dficit C5-C.. menstruacin, embarazo,
auxotipo AHU)

Chlomydio trochomotis, Similar a las UG, pero con menos signos Contacto 7-15 dias. Excluir gonorrea Tetraciclinas o macrlidos
lInIIrItIs no __ Ureoplosmo ureolyticum y sntomas por Gram y cultivo. C. inclusin-
Epldidimitis, proctitis, cervicitis, EIP Giemsa IFD, medios celulares

Tabla 4. Diagnstico diferencial de las uretritis

6
urologia a
Entre los factores que aumentan el riesgo de ITU asociada a catter
Clnica
urinario se pueden enumerar: 1) sexo femenino, 2) edad avanzada, 3)
mala tcnica de sondaje, 41 sistemas de drenaje abiertos y 51 falta de
higiene local. Los hallazgos clnicos son escasos. En el 70% de los pacientes, los sn
tomas son leves. Lo ms frecuente es la aparicin de microhematuria,
Entre los antibiticos disponibles, parece que las quinolonas son los dolor vago en flanco o clico renal. La afectacin vesical, sin embargo,
que mejor eliminan la pelcula biolgica de los catteres infectados, s produce sintomatologa florida con un sndrome cisttico rebelde,
favoreciendo as el tratamiento de la infeccin; en cualquier caso, ste donde la polaquiuria (secundaria a la disminucin de la capacidad
nicamente se recomienda si existe sintomatologa o en el momento de vesical) es lo ms llamativo. En varones, es frecuente la aparicin de
la retirada del catter, por el mayor riesgo de ITU sintomtica y sepsis. una orquiepididimitis crnica que no responde a la terapia habitual.

En el 90% de los pacientes, el anlisis urinario es anormal. Tpicamente


aparece piuria cida con urocultivo negativo. La prueba de laborato
2.4. Tu berc u l os is g e n itou r i n a ri a rio ms importante es el cultivo de M. tuberculosis en medio selectivo
(Lowenstein), ya que los medios de tincin rpida (Ziehl, auramina),
aunque vlidos, pueden dar falsos positivos por contaminacin con M.
Generalmente est ocasionada por Mycobacterium tuberculosis. El apa smegmatis (MtR 02-03, 174).
rato genitourinario es el sitio ms frecuente de afectacin extrapulmonar
(tras la adenitis tuberculosa). Un 5% de los pacientes con tuberculosis
activa presentan afectacin del tracto genitourinario (Figura 2). Diag nstico

Tuberculosis
miliar El cultivo en medio de Lowenstein es positivo en el 90% de los pa
cientes con enfermedad activa, aunque deben obtenerse, al menos, tres
muestras de das diferentes para mejorar la sensibi lidad, ya que el paso
de bacilos a orina no es constante. Actualmente, lo ms rentable es rea
lizar una PCR de orina en busca del ARN del bacilo.

Radiolgicamente, el 90% de los pacientes presentan urogramas altera


Amputacin Pionefrosis dos. El hallazgo ms sugestivo es la presencia de cavidades que comu
catieial nican con el sistema colector. Inicialmente estas cavidades son mnimas
y dan un aspecto "mordisqueado" a los clices. Segn la enfermedad
avanza, pueden encontrarse estenosis infundibulares, ureteropilicas, en
Trompa
Mierovejiga
unin ureterovesical o vejigas pequeas de aspecto rgido. En el punto
ms evolucionado de la enfermedad, el rin puede encontrarse anula
do, disminuido de tamao y con calcificaciones parenquimatosas.

Prstata
y vesfeulas Trata miento
seminales

Estenosis El tratamiento mdico de la enfermedad activa no difiere sustancial


ureteral distal
Epididimitis mente del de la tuberculosis pulmonar en cuanto a frmacos y periodo
de tratamiento. Puede ser necesario el tratamiento quirrgico, depen
diendo de la complicacin asociada, generalmente estenosis de la va
Figura 2. Lesiones de la tuberculosis genitourlnaria excretora e h idronefrosis. En caso de rin no funcionante por lesin
extensa del parnquima, puede ser precisa la nefrectoma.

Tras la inhalacin del bacilo, se produce una diseminacin hematgena


(primoinfeccin) con siembra de bacilos en ambos riones en el 90% de 11 :?n, isoniacida y pirazinamida son los tres antibiticos ms em
los casos. Sin embargo, la enfermedad clnica generalmente es uni lateral. pleados en la tuberculosis.

El periodo de latencia entre la "siembra" y la enfermedad clnica oscila


entre 1 O Y 40 aos, afectando principalmente a pacientes por debajo de
los 50 aos. La lesin inicial microscpica se localiza en los glomru
los en forma de granulomas microscpicos. Al avanzar la enfermedad, 2.5. C istitis intersticial
se produce afectacin ms distal hasta la aparicin de una papilitis
necrotizante, momento en el cual ya puede existir paso de bacilos a la
va excretora donde, por procesos inflamatorios, ocasionar estenosis Aunque no es u n cuadro infeccioso, s e incluye e n el presente captulo
a nivel de los infundbulos caliciales, pelvis y urter, con hidronefrosis esta entidad inflamatoria vesical de origen desconocido. En este senti
secundaria. Las lesiones renales pueden cavitarse y calcificarse, y lle do, se esgrimen dos teoras no demostradas: por un lado, la teora au
gar a producir una destruccin total del parnquima (fenmeno que se toinmunitaria, y por otro, la de un dficit en el recubrimiento urotelial
denomina "rin mastic" ). por glucosaminoglucanos.

7
Manual elO de Medici na y Ciruga, s.a edicin

Cl nica litiasis o tumor vesical) apoyado en los hallazgos cistoscpicos suges


tivos: 1) petequias submucosas, principalmente trigonales, que apare
Suele presentarse en mujeres entre 30 y 70 ar1os, como un cuadro cis cen al distender la vejiga (glomerulaciones), 2) lceras de Hunner. La
ttico crnico en el que destacan disuria, polaquiuria con nicturia y biopsia vesical, adems de descartar la presencia de carcinoma in situ
dolor suprapbico, acompaados en ocasiones de hematuria (20-30%) u otra patologa, revela en algunos casos, un infiltrado intersticial de
(MIR 01-02, 1 06) . mastocitos (MIR 00-01 F, 1 45).

D RECUERDA Trata miento


Existen muchas ms causas de sndrome cisttico: cistitis aguda, tuberculo
sis, carcinoma in situ, etc.

Aunque esta enfermedad raramente supone una amenaza para la vida de


la paciente, su morbilidad es elevada. Desgraciadamente, las diversas al
Diag nstico ternativas de tratamiento nicamente pueden encaminarse a una mejora
sintomtica, en la mayora de los casos con resultados discretos; 1 ) dis
tensin hidrulica vesical, 2) amitriptilina oral, 3) instilacin con dime
El diagnstico es bsicamente por exclusin de otra patologa que tilsulfxido [DMSOJ, 4) corticoides tpicos o sistmicos, 5) denervacin
pueda ocasionM un cuadro similar (infeccin bacteriana, tuberculosis, vesical, 6) cistoplastias e aumento, y en ltimo trmino, 7) cistectoma.

Casos clnicos representativos

Un prosttico, sin otros problemas de salud, portador de sonda uretral pennanente, 4) TBe urogenital.
presenta bacleriuria (> 10' unidades formadoras de colonias) en dos uroculti\los. 5) Pielonefritis crnica por P. aeruginosa.
Cul es la actitud teraputica ms con\lenientel
RC: 4
1 ) Tratamiento antibitico de amplio espectro.
2) Tratamiento antibitico segn antibiograma.
3) Cambio de sonda urinaria exclusivamente. Una paciente de 27 aos acude al servido de Urgencias por dolor en fosa renal
4) Antispticos en vejiga urinaria. derecha, fiebre de 39 "e, escalofros y sndrome micdonal acompaante. Es alrgica
5) Cambio de sond..l urinaria y tratamiento antibitico. a penicilinas. Seale la respuesta correcta:

RC: 3 1) No ser necesario descartar patologa urinaria obstrUd;va en este caso, ya que
presenta un claro sndrome miccional.
2) Para poder hacer el diagnstico de pielonefritis se deber conocer primero 10$
Ante un paciente de 24 aos, que presenta fiebre alta con dolor, inflamacin y datos referidos a la funcin renal.
enrojecimiento testicular izquierdo, cul de las siguientes afirmaciones es INCO 3) Se deber iniciar tratamiento emprico con un -Iactmico.
RRECTA? 4) Si en las primeras horas evoluciona favorablemente podr continuar el tratamien
to de forma ambulante.
1) El diagnstico ms probable es el de epididimitis. 5) El mejor tratamiento disponible es la administracin intramuscular de aminoglu
2) los patgenos ms frecuentcs son Chlamydia lrachomalis y Neisseria gonorrhoeae. csidos.
3) El tratamiento de eleccin es vancomicna + gentamicina.
4) El tratamiento de eleccin puede ser ofloxacino. RC: 4
5) Un tratamiento alternativo es ceftriaxona en monooosis ms 1 o das de doxiciclina.

MIR 99-00, 135; RC: 3 Un paciente de 83 aos sondado de forma permanente acude a la consulta tras
detectrsele dos cultivos positivos lomados con una semana de diferencia. Asegura
encontrarse asintomtico. la acttud ms adecuada ser:
Ante un paciente que presenta febrcula persistente, crisis renoureterales bre\les,
piuria estril, orina con pH cido, microhematuria persistente, con citologa uri 1) Iniciar tratamiento antibitico segn antibiograma de los cuhivO$ obtenidos.
naria negati\la y epiddimos indurados .f.en qu enfermedad se debe pensar pri 2) Tranquilizar al paciente y seguir con su pauta habitual de recambio de sonda.
mero? 3) Realizar cambio de sonda de forma inmediata con tratamiento antibitico.
4) Realizar cambio de sonda de forma inmediata con profila)(is antibitica de 4 das.
1) Sarcoidasis. 5) Retirar la sonda y colocar cistotoma suprapbica.
2) Carcinoma vesical.
3) Carcinoma renal. RC: 2

8
03.
UROLlTIASIS

OrientaCIn

MIR
Tema fundamental en
esta asignatura. Se debe OJ Los clculos ms frecuentes son los de oxalato clcico.
conocer muy bien la actitud
ante la litiasis en general,
ante los distintos tipos de
III Globalmente, la litiasis es ms comn en el varn, salvo las de estruvita, ms comunes en mujeres.
clculos y, especialmente,
todo lo relacionado con m La radiografa de abdomen no permite ver algunos clculos, como los de urato. Sin embargo, la ecografa
puede verlos, independientemente de su composicin.
el tratamiento. Es un tema
rentable y agradecido. as que Litiasis radiotransparentes: Sulfamidas, Indinavir, Urato, Xantinas (SIUX). las de cistina son radiolcldas; y
hay que emplear el tiempo
el resto, radioopacas.
necesario. la tabla-resumen
de urolitiasis puede ser de
las tiazidas son tiles para la hipercalciuria idioptica.
gran ayuda.

los clculos asociados a las resecciones ileales O a la enfermedad inflamatoria intestinal son de oxalato
clcico.
Precipitan en medio cido: cido rico y cistina. Precipitan en medio alcalino las que contienen fosfatos
(fosfato amnico magnsico o estruvita, y el fosfato clcico).

En el tratamiento de la litiasis por cido rico es beneficioso alcalinizar la orina.

los clculos de oxalato NO se ven alterados por el pH (al Oxal, el pH de la igual).

los clculos de estruvita se relacionan con microorganismos productores de ureasa, como Proteus.

las contraindicaciones absolutas para la LEOC son: embarazo, infeccin activa y obstruccin de las vas
urinarias distal al clculo.

3 . 1 . E p id e m iologa
Son numerosas las sustancias que se han identificado formando parte de 105 clculos. Su i ncidencia vara segn
el pas, e incluso segn las reas geogrficas dentro del mismo pas.

Se pueden distinguir seis grupos de componentes:


Oxalato clcico.
Fosfato clcico.
Fosfato no clcico.
Compuestos purnicos (cido rico, urato amnico, urato sdico, xantina, 2 , 8 dihidroxiadenina).
Aminocidos (cistina).
Otros (carbonato clcico, sulfamidas, etc.).

[?J Preguntas
Los clculos de oxalato clcico son los ms frecuentes, con cifras en torno al 65%, seguidos por los infectivos
-MIR 08-09, 93 y cido rico (alrededor del 15% cada uno). Fosfato clcico un 5%, y los de cistina con una incidencia baja
- MIR 06-07, 93, 106
(1-3%).
-MIR 05-06, 104
-MIR 04-05, 104
- MIR 03-04, 44, 80 La tercera dcada es la edad media de aparicin, por primera vez, de la litiasis salvo en los de cistina, que suelen
- MIR 02-03, 1 76
- MIR OO-01 , 1 1 7
ser de aparicin ms prematura.
- MIR 99-00, 1 89
- MIR 99-00F, 143, 146
- MIR 98-99, 135, 138
En Espaa, la incidencia de litiasis alcanza al 4,2% de la poblacin, con mayor afectacin de varones que mu
- MIR 98-99F, 144 jeres. nicamente los clculos infectivos tienen mayor incidencia en la mujer.

9
03.
UROLlTIASIS

OrientaCin Aspectos esenciales

MIR
TerT'kl fundamental en
esta asignatura. Se debe los clculos ms frecuentes son los de oxalato clcico.
conocer muy bien la actitud
ante la litiasis en general. Globalmente, la litiasis es ms comn en el varn, salvo las de estruvita, ms comunes en mujeres.
ante los distintos tijX)S de
clculos y. especialmente, la radiografa de abdomen no permite ver algunos clculos, como los de urato. Sin embargo, la ecografa
todo lo relacionado con
puede verlos, independientemente de su composicin.
el tratamiento. Es un tema
rentable y agradecido, as que Litiasis radiotransparentes: Sulfamidas, Indinavir, Urato, Xantinas (SIUX). las de cistina son radiolcidas; y
hay que emplear el tiempo
el resto, radioopacas.
necesario. La tabla-resumen
de urolitiasis puede ser de
las tiazidas son tiles para la hipercalciuria idioptica.
gran ayuda.

los clculos asociados a las resecciones ileales o a la enfermedad inflamatoria intestinal son de oxalato
clcico.

o Precipitan en medio cido: cido rico y cistina. Precipitan en medio alcalino las que contienen fosfatos
(fosfato amnico magnsico O estruvita, y el fosfato clcico).

En el tratamiento de la litiasis por cido rico es beneficioso alcalinizar la orina.

los clculos de oxalato NO se ven alterados por el pH (al Oxal, el pH de la igual).

los clculos de estruvita se relacionan con microorganismos productores de ureasa, como Proteus.

Las contraindicaciones absolutas para la LEOC son: embarazo, infeccin activa y obstruccin de las vas
urinarias distal al clculo.

3 . 1 . E p i d e m iologa
Son numerosas las sustancias que se han identificado formando parte de los clculos. Su incidencia vara segn
el pas, e incluso segn las reas geogrficas dentro del mismo pas.

Se pueden distinguir seis grupos de componentes:


Oxalato clcico.
Fosfato clcico.
Fosfato no clcico.
Compuestos purnicos (cido rico, urato amnico, urato sdico, xantina, 2,8 dihidroxiadenina).
Aminocidos (cistina).
Otros (carbonato clcico, sulfamidas, ete.).
U Preguntas
Los clculos de oxalato clcico son los ms frecuentes, con cifras en torno al 65%, seguidos por los i nfectivos
- MIR 08-09, 9 3 y cido rico (alrededor del 1 5% cada uno). Fosfato clcico un 5%, y los de cistina con una i ncidencia baja
- MIR 06-07, lJ3. 106
( 1 -3%).
- MIR 05-06, 104
- MIR 04-05, 104
- MIR 03-04, 44, 80 La tercera dcada es la edad media de aparicin, por primera vez, de la litiasis salvo en los de cistina, que suelen
- MIR 02-OJ, ' 76
- MIR OO-Ol , 1 1 7 ser de aparicin ms prematura.
- MIR 99-00, 189
- MIR 99-00F, 143, 1 4 6
En Espaa, la incidencia de l itiasis alcanza al 4,2% de la poblacin, con mayor afectacin de varones que mu
- MIR 98-99, 1 3 5 , 1 38
- MIR 98-99F, 144 jeres. nicamente los clculos infectivos tienen mayor incidencia en la mujer.

9
Manual eTO de Medicina y Ciruga, B.a edicin

o RECUERDA coraliforme o "en asta de venado" [Figura 4]), manifestndose no como


clico, sino como infecciones urinarias de repeticin, dolor lumbar
las infecciones urinarias son ms frecuentes en mujeres que en varones.
Por eso los clculos de estruvita tambin lo son. sordo, hematuria o incluso insuficiencia renal terminal.

La enfermedad litisica recidiva en el 40% de los casos, con una media


de un nuevo clculo cada dos o tres aos. Por recidiva se entiende la apa
ricin de una nueva litiasis de la misma composicin y en la misma lo
calizacin, en un intervalo menor de cuatro aos entre un clculo y otro.

3.2. M a n ifestaciones c l n i cas


y su manejo a g u d o

El dolor agudo del clico renal es l a manifestacin ms tpica de la li


tiasis renal. El dolor se produce por la sobredistensin de la va urinaria
tras la obstruccin de sta por el clculo. Es lgico, por tanto, que el
clculo deba desplazarse desde su origen calicial para producir sinto
matologa aguda. Ocasionalmente se observan cuadros de dolor vago
renal en relacin con litiasis caliciales no desplazadas.
Diagnstico
El clico renal o crisis renoureteral suele aparecer de forma progresi
va sobre la fosa lumbar afectada, irradindose por el flanco hacia la
ingle y los genitales (Figura 3). El paciente generalmente se encuentra El anlisis bsico de orina muestra generalmente hematuria y leuco
afectado, con dolor que no cede con reposo, por lo que cambia de cituria. Una piuria importante apoyara la posibil idad de infeccin
postura continuamente. Puede acompaarse de un cortejo vegetativo sobreaadida, aunque ninguno de estos datos es realmente determi
con nuseas, vmitos y sudoracin. El dolor irradiado hacia la ingle nante.
generalmente indica que el clculo ha alcanzado el urter. Cuando se
encuentra en vecindad de la vejiga, o bien dentro de sta, puede apare Los cristales de oxalato clcico d i h idralado aparecen como bipir
cer un cuadro irritativo, similar al sndrome miccional con polaquiuria, mides tetragonales a l observarlos con lupa b i nocular. Los de oxa
disuria y tenesmo vesical. lato clcico monohidratado aparecen como cristales alargados que
adoptan forma de empalizada, formando clculos de estructura ra
diada, con aspecto compacto y macizo. Entre los fosfatos clcicos,
la brushita es el compuesto ms cido, formando cristales grandes
Sudoracin en forma de abanico de color azul con luz polarizada. Las apatitas
tienen aspecto microgranular o esferoctico. El cido rico aparece
bajo la lupa como una desordenada aglomeracin de cristales. En
algunos clculos, los cristales estn tan juntos que se asemejan a u n a
masa continua.

La estruvita (o fosfato amnico magnsico) es el componente ms ca


RX simple de abdomen
racterstico de los clculos producidos por infeccin por grmenes
con clculo a nivel de l3
urealticos. Sus cristales tienen formas prismticas polimorfas, y rara
mente se observan los cristales "en atad" que pueden hal larse en el
sedimento. La cistina se reconoce fcilmente por su aspecto acarame
lado, formando cristales hexagonales en prismas o lminas.
Dolor de inicio
En teora, el 90% de los clculos son visibles en una radiografa simple
de abdomen, aunque este porcentaje es considerablemente menor en

Dolor irradiado Sndrome miccional


las radiografas urgentes sin preparacin intestinal. Radiolgicamente,
Hematuria la mayora de los clculos son radioopacos, exceptuando los de cido
rico y algunas otras composiciones infrecuentes (sulfamidas, xantina,
indinavir) (MIR 04-05, 1 04).

El estudio de imagen se completar mediante otras tcnicas diag


nsticas. La ecografa permitir visualizar incluso las l itiasis radio
Figura 3. Diagnstico de urolitiasis
transparentes, con el inconveniente de no ser vistas aquellas ubi
cadas en el trayecto ureteral (salvo las zonas cercanas a la vejiga o
Los clculos infectivos de estruvita, y en menor medida, los de cido al rin). Tambin se podr evaluar el grado de hidronefrosis ( M I R
rico y cistina pueden crecer modelando las cavidades renales ( l itiasis 99-00, 1 89).

10
urolOgia
La urograffa ofrece informacin morfolgica y funcional de ambos
riones (Figura 5). Debe tenerse en cuenta que, durante el clico
renal, puede observarse una anulacin funcional, sin que signifi
que necesariamente deterioro de dicha unidad renal. Mediante
esta tcnica se puede diagnosticar todo tipo de clculos, ya sean
radiotransparentes o radioopacos. El principal inconveniente de
este procedimiento es la introduccin de contraste yodado, que est
contra indicado en los pacientes con alergia, creatin i na mayor de 2,
mieloma mltiple o deshidratacin i mportante.

Segn las guas clnicas, la urografa intravenosa (UIV) actualmente ha


sido desplazada por la Te helicoidal sin contraste, que se ha convertido
en el nuevo estudio de referencia para las litiasis. Aunque su alto coste
hace que todava no est extendido su uso, permite evaluar todo tipo
de clculos.
Figura 6. Doble J derecho. litiasis ureteral derecha. litiasis coraliforme izquierda

Tratamiento

3.3. Eva l u acin y trata m iento


El manejo agudo del clico renal se basa en el control del dolor.
Para esto, es preciso conseguir una disminucin de la presin den de l a l itiasis ren a l
tro de la va urinaria, Jo que puede hacerse, sobre todo, con anti
inflamatorios, que disminuyen el dolor y la d iuresis a l i n hibir la
sntesis de prostaglandinas. Asimismo, se pueden usar espasmolti Este apartado se puede dividir en dos partes. Por u n lado, el estudio
cos, que disminuyen la presin intraureteral al relajar la pared del de la litiasis con la finalidad de instaurar un tratamiento preventivo
urter. de su formacin, y por otro, el estudio y tratamiento de la litiasis ya
formada.
Existen una serie de situaciones en las que el clico renal se convierte
en una urgencia que precisa de hospitalizacin, y eventualmente, de
manipulacin invasiva: Estudio
Obstruccin grave, principalmente si se acompaa de litiasis mayor
y tratamiento preventivo
de 1 0 mm.
Fiebre elevada (mayor de 38 OC).
Dolor incontrolable.
Rin nico. La evaluacin del paciente con litiasis se basa en un estudio meta
blico para determinar qu factores son modificables, en un intento
Asimismo, en pacientes diabticos, por el mayor riesgo de com de evitar la recidiva (Tabla 5).
pl icaciones, es aconsejable, si no el i ngreso, a l menos una obser

a RECUERDA
vacin estricta. Una situacin s i m i l a r ocurre durante el emba razo, Este estudio debe reservarse para aque
donde una dilatacin leve de la va urinaria puede considerarse l l os pacientes con alta proba b i l idad de
la furosemida, al re
" fisiolgica", pero obstrucciones ms importantes o la aparicin recidiva, aunque cada vez ms autores vs que las tiazidas,
de fiebre hacen aconsejable l a colocacin de u n catter u reteral indican que debe realizarse a todos Jos aumenta el calcio uri
nario.
(Figura 6). pacientes (Tabla 6).

11
Manual CTO de Medicina y Ciruga, B.a edicin

LITIASIS plante heptico, que suele ir unido al renal, aunque algunos casos
LITIASIS CALCICA LITIASIS LITIASIS
INFECCIOSA responden a piridoxina.
(OXALATO O FOSFATO) URICA CtSTINICA
(ESTRUVITA)

11 RECUERDA
Hipercalciuria idioptica Gota primaria . Cistinuria . Infecciones
Hipercalciuria secundaria Hemopatas por grmenes
a hipercalcemia Enfermedades productores La causa ms frecuente de hipercalcemia en un pac iente ambulatorio
Hiperuricosuria digestivas de ureasa es el hiperparatiroidismo primario. En cambio, la hipercalcemia ms
Hiperoxaluria Ingesta excesiva frecuente en uno ingresado es la de origen neoplsico.
Hipocitraturia de purinas
Acidosis renal tubular Frmacos
distal litiasis rica
No obstante, la mayora de los casos de hiperoxaluria son secun
litiasis clcica idioptica idioptica darios a malabsorcin de cidos grasos por enfermedades crnicas
pancreatobiliares, derivacin intestinal para el tratamiento de la
Tabla 5. Tipos de litiasis. Situaciones que favorecen su aparicin
obesidad mrbida, reseccin ileal, enfermedad inflamatoria intes
tinal (MIR 05-06, 1 04; MIR 00-0 1 , 1 1 7), hipercalciuria coincidente
Edad temprana de aparicin o por falta de calcio en la dieta, lo que permite que exista mayor
Litiasis bilateral cantidad de oxalato intestinal para su absorcin.
litiasis en rin nico o malformado
La intoxicacin por etilenglicol y metoxiflurano puede producir hi
Composicin poco frecuente
litiasis recidivante
peroxaluria, as como la ingesta de vitamina C en altas dosis. En
Nefrocalcinosis todos estos casos secundarios, el tratamiento con colestiramina, una
Litiasis coraliforme dieta pobre en grasas y la correccin de la malabsorcin, en la me
Tabla 6. Pacientes con indicacin de estudio metablico dida de lo posible, suelen ser medidas eficaces.
Hipocitraturia. Excrecin de citrato inferior a 300 mgl24 h. Gene
ralmente se asocia a otras anomalas urinarias. Aunque de causa
Desde el punto de vista prctico, las litiasis se pueden dividir en: las desconocida, puede contribuir una dieta rica en protenas, hipoca
de composicin clcica y las de otras composiciones, ya que el primer liemia, enfermedad intestinal o infeccin urinaria.
grupo supone la mayora de los casos (70-80%) tratados habitualmente. Hiperparatiroidismo primario. Supone la causa ms frecuente de
hipercalciuria conocida (vase Seccin de Endocrinologa, metabo
lismo y nutricin) (MIR 98-99F, 1 44).
Litiasis clcica Acidosis tubular renal distal (vase Seccin de Nefrologa). Enfer
medad autosmica recesiva. Consiste en la imposibilidad del tbulo
En la mayora de las ocasiones se desconoce el origen de la litiasis cl distal para excretar hidrogeniones a la orina (orinas persistentemen
cica, aunque se puede hacer una aproximacin a los factores de riesgo te alcalinas) con aumento de la eliminacin de calcio a la orina.
que influyen en su aparicin. Slo en un pequeo porcentaje de casos Existen formas incompletas que se observan en pacientes formado
existe una enfermedad de base que puede ser tratada, y de esta forma res de clculos de oxalato clcico y con hipercalciuria idioptica.
desaparece la formacin de clculos clcicos. En stos, probablemente la acidosis tubular no juegue un papel im
Hipercalciuria idioptica. Es la causa ms frecuente de litiasis cl portante y responden a tiazidas (MIR 98-99, 1 38).
cica. Se define como una excrecin urinaria de calcio mayor de Otras circunstancias que favorecen la litiasis clcica son: sarcoido
300 mgl24 h en el varn y 250 mgl24 h en la mujer. De cara a su sis, sndrome de Cushing, diuresis escasa, dficit de inhibidores o
manejo, las tiazidas disminuyen el calcio urinario, reduciendo la anomalas en el pH urinario (alcalosis).
formacin de litiasis (MIR 03-04, 44; MIR 99-00F, 1 46; MIR 98- Litiasis clcica idioptica. Aproximadamente en el 20% de los pa
99, 1 35). La administracin de citrato potsico ayuda a evitar la cientes con litiasis clcica no se demuestra ninguna anomala en el
hipopotasemia y aumenta el citrato urinario, que es inhibidor de la estudio metablico.
litognesis (Tabla 7).

I Litiasis rica
ABSORTIVAS RESORTIVAS RENALES
,
Aporte excesivo Hiperparatiroidismo Acidosis tubular
El cido rico no disociado es poco soluble en orina. Con un pH uri

Sd. de Surnen (leche Inmovilizacin distal


y alcalinos ) Tumorales Idioptca nario de 5, la solubilidad del cido rico es nicamente de 1 00 mgll,
Hipervitamlnosls O Enf. Paget mientras que con un pH de 7 es de 1 .580 mgll. Esto demuestra la gran
Idioptica Sd. de Cushing importancia del pH urinario en la formacin de clculos de cido ri
Sarcoidosis
co. Aparte de stos, tambin existe una pequea proporcin de clcu
Tabla 7. Causas ms frecuentes de hipercakiuria los de urato monosdico y urato amnico.

Hiperuricosuria. Excrecin en orina de ms d e 800 mgl24 h en el El objetivo del tratamiento es reducir el cido rico excretado y au
varn o 750 mgl24 h en la mujer. Adems de favorecer la litiasis mentar el pH urinario (MI R 03-04, 80), ya que los clculos ms fre
rica, la hiperuricosuria constituye un factor de riesgo para la for cuentes en pacientes hiperuricmicos son los de cido rico. Por otra
macin de clculos de calcio, probablemente por nucJeacin hete parle, este tipo de clculos son los que mejor responden al tratamien
rognea sobre ncleos de cido rico o urato sdico. Generalmente to mdico mediante quimilisis por alcalin izacin urinaria. Pueden
se debe a un exceso de purinas en la dieta. administrarse diversos lcalis; el citrato potsico impedira el teri
Hiperoxaluria. Se considera como tal la excrecin en orina de ms co riesgo de formacin de clculos clcicos por su efecto inhi bidor,
de 40 mgl24 h. Existe una hiperoxaluria primaria, que es conse pero tambin pueden tratarse con bicarbonato sdico o citrato sdi
cuencia de un defecto enzimtico autosmico recesivo; no tiene CO (MIR 02-03, 1 76). Una a lternativa es la acetazolamida en dosis de
tratamienlo y generalmente conduce a insuficiencia renal por litiasis 250 mglda. Cuando, adems, la uricemia es alta, puede tratarse con
recidivante. El nico tratamiento que existe actualmente es el tras- alopurinol (MIR 06-07, 93).

12
urolOgia a
Litiasis cistnica complejo enzima-inhibidor irreversible. Se utilizan bsicamente dos
sustancias de esta naturaleza: el cido propinico y el acetohidroxmi
La cistinuria es un trastorno autosmico recesivo en el que existe un co. Su empleo suele venir acompaado de cefaleas, temblores, trom
defecto de absorcin, a nivel intestinal y tubular proximal, de los ami bosis venosas u otros sntomas neurolgicos, por lo que tampoco son
nocidos dibsicos: cistina, ornitina, lisina y arginina (COLA), aunque de gran aceptacin.
parece que puede existir un trastorno en el que nicamente se ve afec
tada la cistina, lo que i ndicara que, adems de un mecanismo de trans Todo lo relativo al estudio de la nefrolitiasis expuesto anteriormente se
porte comn, existe uno independiente para la cistina. puede repasar en la Tabla 8.

Los niveles de cistina en orina de 24 horas son superiores a 1 00 mg, de


hecho, los homoeigotos pueden excretar ms de 600 mglda. El diag Tratamiento de la litiasis ya formada (Figura 7)
nstico se realiza identificando los caractersticos cristales hexagonales
en orina, o por una prueba positiva de nitroprusiato sdico (la orina se
tie de azul en pacientes afectados por esta enfermedad: test de Brand). Los clculos ya formados no expulsables (> 4-5 mm) precisan de
tratamiento "agresivo", es decir, necesitan ser extrados qui rrgica
El tratamiento consiste en aumentar la diuresis diaria (ms de 3 I/da), mente o fragmentados de forma que puedan ser expu lsados espon
alcalinizar la orina por encima de 7,5 y, en caso de que esto sea insufi tneamente.
ciente, puede iniciarse tratamiento con D-penicilamina (250 mgl6 h) o
a-mereaptopropionilglieina (250 mgl6 h). A continuacin se analizan brevemente las diversas formas de tratamiento.
Ciruga. Ha sido el tratamiento estndar hasta la aparicin de la
litotricia extracorprea. An hoy da, es preciso recurrir a la ciruga
Litiasis infectiva cuando fracasan las ondas de choque o en determinados casos para
reducir la masa litisica (clculos coraliformes).
Los clculos infectivos de estruvita o de fosfato amnico magnsico (Mg Endourologa. La manipulacin endoscpica de la va urinaria es
NHl04-6H20) se desarrollan en un ambiente alcalino, producido por cada da ms accesible gracias a las mejoras tcnicas. Puede reali
infeccin persistente de grmenes que hidrolizan la urea, aumentando zarse extraccin directa del clculo mediante diversos tipos de pin
la cantidad de amonio urinario. Los principales grmenes que poseen zas o cestillas, o bien fragmentar previamente el clculo mediante
ureasa, adems de diversas especies de Proteus (MIR 06-07, 1 06). son diversas fuentes de energa, como la electrohidrulica, ultrasnica o
Pseudomonas, Klebsiella, Serratia y Enlerobacter. La presencia de cuer lser. Se puede acceder hasta el clculo mediante ureterorrenosco
pos extraos (sondas vesicales, suturas) favorece su formacin. pia (U RS) o nefrolitotoma pereutnea ( NLPC).
litotricia extracorprea por ondas de choque (LEOC). Las ondas de
Para su tratamiento se han empleado diversos mtodos, generalmente choque se transmiten a travs de los tej idos corporales con la misma
ineficaces. La antibioterapia nicamente mantiene estril la orina du impedancia acstica que el agua hasta alcanzar la litiasis, sobre la
rante los cursos de tratamiento. que produce fenmenos de compresin y descompresin que con
ducirn a su fragmentacin.
Parece ms prometedor el uso de inhibidores de la ureasa con cidos Prcticamente todos los clculos son susceptibles de tratamiento me
hidroxmicos. Estos son molculas anlogas a la urea que forman un diante LEOe. La nica limitacin seran aquellos clculos no localiza-

ESTRUVITA (FOSFATO
SALES CALCICAS AClDO URICO CISTlNA
AMNICO MAGNSICO)

. Oxalato clcico: 55-60% 10-15% 5-10% 1 -3%


Frecuencia
Fosfato clcico: 1 0-' 5%

Sexo Varn Mujer Varn Varn = Mujer

Hipercalciuria idioptica Infeccin por grmenes ureasa (+) Gota (50%) Cistinuria
Etiologia Idioptica . Idioptica 50%)
Hiperuricosuria (20%) . Hiperuricemias secundarias

pH Alcalino Alcalino cido cido

Radiologia Radioopacos Radioopacos Radiotransparentes Rad iolcidos


Formas prismticas polim6rficas Aglomerados de cristales Cristales hexagonales en prismas

:liD
Cristales-en atad- desorganizados, a veces formando o lminas
masas continuas

q &o 0
W
OO
Cristales de fostato Ca
Moologa

O rfF
de los cristales

1lJ
1Iil' -ft>

D fSJ
Cristales de OxCa Cristales de estruvita Cristales de cido rico Cristales de cistina

Hipercalciuria idioptica: tiazidas cido propi6nico y cido Alcalinizar la orina Forzar diuresis (ingesta hdrical
Hiperoxaluria 1 .11; piridoxina acetohidroxmico Alopurinol (si hay hiperuricemia) Alcalinizar orina
Tratamiento
Hiperoxaluria 2.11; colestiramina Antibioterapia Dieta de bajo contenido D-penicilamina (si no hay
En ocasiones ciruga proteico respuesta)

Tabla 8. Tabla-resumen de las nefrolitiasis

13
Manual CTO de Medicina y Ciruga, s.a edicin

de su tamao, composicin y
dureza, localizacin, particula
MANEJO DE LA UROLlTIASIS
ridades anatmicas de la va ex
cretora y paciente, funcin renal
y tipo de litotriptor disponible.
Crisis Estable
CUADRO AGUDO CUADRO CRNICO
La presencia de hipertensin

/
Edad?
Indican o arterial no controlada facilita
Tipo de clculos?
no LEOC o
ciruga
Periodicidad de la clnica? el riesgo de hemorragia duran

No
comlicado - --
Complicado Tipo de sntomas?
te la sesin de litotricia, luego
Viabilidad renal?
deber ser estabil izada previa
TTO. SINTOMATICO Obstruccin gr ave
mente a la misma y constituye,
Tratar la condicin

Espasmolticos Infeccin, fiebre litotricia Ciruga en c ierto modo, por ello, con
preexistente
y antiinflamatorios Dolor incoercible
t t t

traindicacin relativa de LEOC
Reposo e hidratacin Rin nico

(MIR 08-09, 93; M I R 99-00F,


CALCICA EXTRACORPREA URETEROTOMIA 1 43 ) (Tabla 9).
acidificar (lEOCJ PIELOllTOTOMIA
Ecografa renal
(no til si oxalato), PERCUTANEA NEFRECTOMIA

citratos, tiazidas ultrasonogrfica


colestiramina con microlumbotoma Complicaciones
Dilatacin
y dieta baja en ENOOSCPICA
INGRESO YTTO. AGRESIVO grasas y rica va ureteral
DRENAJE en calcio, si La expulsin de fragmentos Ii
(catter o nefrostoma) hiperoxaluria CONTRAINDICADA EN tisicos puede ocasionar un
no. PARENTERAL RICA EMBARAZO

alcalinizar,
clico renal y, con menor fre
Antibiticos INFECCIN
Remontar hemodinmica alopu rino l OBSTRUCCIN DISTAL cuencia, obstruccin ureteral
Equilibrio electroltico ESTRUVITA Aneurismas (sleinstrasse o "calle litisica/l).
acetohid rox m ico

Narcticos Coagulopatfas
VIGILANCIA ESTRECHA CISTINA Obesidad
O-penicilamina,

Arritmia cardaca Esta posib i l idad es mayor ante


vit. B y alcalinizar litiasis de gran tamao, por lo
que en algunos de estos casos
Figura 7. Manejo de la litiasis renal ya formada se puede colocar un catter
de derivacin urinaria (ne
frostoma o doble J) antes de
bIes por su pequeo tamao 2-5 mm). Cualquier litiasis podra ser la LEOC para d i s m i n u i r este riesgo, generalmente en litiasis supe
tratada con LEOC, aunque esto tendr que ser matizado en funcin riores a 2 c m .

RElATIVAS (PRECISAN DE CONTROL PREVIO


Derivadas del efecto directo de las ondas de choque, pueden aparecer
ABSOLUTAS
AL TRATAMIENTO) contusiones renales manifestadas como hematuria, hematomas rena
Embarazo Alteraciones de la coagulacin
les, equimosis o eritema cutneo, y en grado mximo, rotura renal. La
Obstruccin distal Aneurisma artico hematuria se considera la complicacin ms frecuente de la litotricia.
Infeccin activa Alteraciones del ritmo cardaco, marcapasos
o desfibriladores
Ms controvertida es la terica relacin de la LEOC con la aparicin de
Obesidad
Hipertensin arterial descontrolada hipertensin arterial, ya que no est demostrada en las ltimas revisiones
publ icadas, aunque s la relacin entre hematoma renal post-LEOC e
Tabla 9. Contraindicaciones de la LEOC (MIR 99-00F, 143) hipertensin arterial.

A un hombre de 29 aos, con antecedentes de dolor tipo clico en fosa renal izquierda das de evolucin, asociado en las ltimas 24 horas a fiebre, escalofros y malestar
que cedi con tratamiento analgsico, se le practica una urografa intravenosa, apre general. Analtica de sangre: plaquetopenia, leucocitosis y disminucin de la activi
cindose defecto de replecin radiotransparente de 6x7 mm en tercio distal de urter dad de la protrombina. Analtica de orina normal. Radiografa de abdomen con claras
izquierdo. El pH de la orina fue de 5,5; asimismo, se observan cristales de urato, 9-12 imgenes de litiasis. Eco renal: dilatacin moderada de sistema excretor izquierdo.
hemates por campo y escasa leucocituria. Cul sera el tratamiento ms apropiadol Cul es la conducta ms adecuada?

1 ) Alopurinol va oral. 1 ) Solicitar hemocultivos y urocultivo para establecer la necesidad de antibiote


2) Ureteroscopia con extraccin del clculo. ra p i a.
3) Nefrolitotoma endosc6pica percutnea. 2) Realizar urografa intravenosa para intentar filiar la causa.
4) Alcalinizacin de la orina por va oral. 3) Hidratar a la paciente bajo observacin rigurosa, y repetir ecografa a l as 48
5) Administracin de D-penicilamina. horas.
4) Colocar catter doble) o practicar nefrostoma percutnea de forma inmediata
MIR 03-04, 80; RC: 4 con cobertura antibitica.
5) laparotoma exploradora para objetivar causa, y realizar tratamiento antes de que
el cuadro est muy evolucionado.
Mujer de 50 aos, diabtica insulinodependiente, con infecciones urinarias y clicos
nefrticos de repeticin. Acude a Urgencias por dolor en fosa renal izquierda de cinco RC: 4

14
04.
TUMORES RENALES

OnentaCln Aspectos esenciales

MIR
El aclenocarcinoma fenal es un
tema "de moda" en el examen (jJ El ms frecuente de los tumores slidos renales es el hipernefroma.
MIR. Cualquier aspecto de
este lema puede aparecer,
pero es fundamental reconocer
lIJ El paciente caracterstico es un varn de mediana edad, obeso y fumador.
el sndrome de $Iauffer,
que ha sido recientemente GJ la trada clsica consiste en hematuria, dolor y masa en flanco, actualmente, lo ms habitual es que sea
incidentaloma (asintomtico). Si produce sntomas, el ms frecuente es la hematuria.
introducido y preguntado
varias veces consecutivas. Hay que sospechar tumor renal ante un varicocele izquierdo, de aparicin sbita y que no cede con el
decbito.

El hipernefroma puede producir multitud de sndromes paraneoplsicos. Esto puede complicar bastante el
diagnstico, de ah el sobrenombre de "tumor del internista".

La elevacin de las transaminasas sin afectacin heptica es tpica del hipernefroma (sndrome de Stauffer).

No se debe confundir un quiste simple con un hipernefroma. los criterios de quiste simple son: contorno
liso, contenido transnico y refuerzo posterior.

la primera prueba de imagen, ante la sospecha de hipernefroma, sera la ecografa.

El tratamiento fundamental del hipernefroma es la extirpacin quirrgica. La quimioterapia y radioterapia


tienen un papel muy secundario.

4.1 . Ca rc inoma de c l u las rena les


(adenoca rc inoma re nal, h ipernefroma)

Es e l tumor slido renal ms frecuente (90%) (Figura 8) (MIR 99-00, 1 77). Es un tumor fundamentalmente d e l a
edad adulta, con mayor incidencia entre los 4 0 y 60 aos, con predominio e n e l varn 2 : 1 a excepcin d e la
variedad cromfoba, tpica de las mujeres.

Entre los factores de riesgo que se han implicado se encuentran el humo del tabaco, el cadmio y la obesidad.
Existen formas familiares que suelen ser mltiples y bilaterales, como en la enfermedad de Von Hippellindau y,
en menor medida, la esclerosis tuberosa (MIR 0001 , 1 20). Se han identificado a lteraciones cromosmicas que
implican al cromosoma 3.

D RECUERDA Asimismo, existe una incidencia aumentada en el rin poliqustico,


en la enfermedad qustica adquirida de la insuficiencia renal crnica
la esclerosis tuberosa y la enferme
dad Von Hippel.lindau se asocian y en los riones malformados, como el rin "en herradura". Procede
tambin a otro tumor renal, el an de las clulas del tbulo contorneado proximal, y microscpica mente
IIJ Preguntas giomiolipoma.
predominan las clulas claras sobre las granulares y fusiformes .

MIR 09-10, 99
- MIR 06-07, 102
- MIR 05-06, 105 Presentacin
- MIR 04-05, 105
- MIR 01-02, 109
- MIR 00-0 1 , ' 2 0
- M I R 99-00, 1 7 7
- MIR 99-00F, 144
La trada clsica: hematuria dolor y masa en el flanco ocurre nicamente en el 1 0% de los casos y, cuando se
- MIR 98-99F, 146 presenta as, generalmente se trata de una enfermedad avanzada.

15
Manual eTO de Medicina y Ciruga, B.a edicin

de su tamao, composicin y
dureza, localizacin, particula
MANEJO DE LA UROllTIASIS
ridades anatmicas de la va ex
cretora y paciente, funcin renal
y tipo de l i lotriptor disponible.
Crisis Estable
CUADRO AGUDO CUADRO CRNICO
La presencia de h ipertensin

/
Edad?
Indican o arterial no controlada facilita
Tipo de clculos?
no LEO( o
Periodicidad de la clnica? el riesgo de hemorragia duran
- - ciruga
Compl icado Tipo de slntomas?
te la sesin de litotricia, luego
t
Viabilidad renal?
deber ser estabil izada previa
nO. SINTOMATICO Obstruccin grave
mente a la misma y constituye,
Tratar la cond icin

Espasmolfticos . Infeccin, fiebre litotricia Ciruga en cierto modo, por ello, con
p reexistente
y a ntiinflamatorios Dolor incoercible
t t t

traindicacin relativa de LEOC
Reposo e hidratacin Rin nico
(MIR 08-09, 93; M I R 99-00F,
CALCICA EXTRACORPREA URETEROTOMIA 1 43) (Tabla 9).
acidificar (LEOCI PIELOLITOTOMIA
Ecografa renal
(no til si oxalato), PERCUTANEA NEfRECTOMIA

citratos, tiazidas ultrasonogrfica


colestiramina con microlumbotoma Complicaciones
Dilatacin y dieta baja en ENOOSCPICA

INGRESO VITO. AGRESIVO grasas y rica va ureteral


DRENAJE
en calcio, si La expulsin de fragmentos Ii
(catter o nefrostoma) hi peroxal uria CONTRAINDICADA EN tisicos puede ocasionar un
RICA EMBARAZO
no. PARENTERAL
clico renal y, con menor fre
Antibiticos alcali nizar, INFECCIN
cuencia, obstruccin ureteral

Remontar hemodinmica alopurinol OBSTRUCCIN DISTAL


ESTRUVITA Aneurismas (steinstrasse o "calle litisica").
Equi librio electroltico

Narcticos acetohidroxmico Coagulopatfas


VIGILANCIA ESTRECHA CISTlNA Obesidad
Esta posibilidad es mayor ante

D-penici lam i na, Arritmia cardiaca


vit. B6 y alcalinizar litiasis de gran tamao, por lo
que en algunos de estos casos
Fig ura 7. Manejo de la litiasis renal ya formada
se puede colocar un catter
de derivacin urinaria (ne
frosloma o doble J) antes de
bies por su pequeo tamao 2-5 mm). Cualquier litiasis podra ser la LEOC para dismi n u i r este riesgo, generalmente en litiasis supe
tratada con LEOC, aunque esto tendr que ser matizado en funcin riores a 2 cm.

RelATIVAS (PRECISAN DE CONTROL PREVIO


Derivadas del efecto directo de las ondas de choque, pueden aparecer
ABSOLUTAS
Al TRATAMIENTO) contusiones renales manifestadas como hematuria, hematomas rena

Embarazo
les, equimosis O eritema cutneo, y en grado mximo, rotura renal. La
Alteraciones de la coaguladn
Obstruccin distal Aneurisma artico hematuria se considera la complicacin ms frecuente de la litotricia.
I nfeccin activa Alteraciones del ritmo cardiaco, marca pasos
desfibriladores
Ms controvertida es la terica relacin de la LEOC con la aparicin de
Obesidad
Hipertensin arterial descontrolada
hipertensin arterial, ya que no est demostrada en las ltimas revisiones
publicadas, aunque s la relacin entre hematoma renal post-LEOC e
Tabla 9. Contraindicaciones de la LEOC (MIR 99-OOF, 143) hipertensin arterial.

A un hombre de 29 aos, con antecedentes de dolor tipo clico en fosa renal izquierda das de evolucin, asociado en las llimas 24 horas a fiebre, escalofros y malestar
que cedi con tratamiento analgsico, se le practica una urografa intravenosa, apre. general. Analtica de sangre: plaquetopenia, leucocitosis y disminucin de la activi
cindose defecto de replecin radiotransparenle de 6x7 mm en tercio distal de urter dad de la protrombina. Analtica de orina normal. Radiografia de abdomen con daras
izquierdo. El pH de la orina fue de 5,5; asimismo, se observan cristales de urato, 9-12 imgenes de litiasis. Eco renal: dilatacin moderada de sistema excretor izquierdo.
hemates por campo y escasa leucocituria. Cul sera el tratamiento ms apropiado? Cul es la conducta ms adecuadal

1 ) Al opurinol va ora l. 1 ) Solicitar hemocu lti vos y urocultivo para cstablecer la necesidad de antibiotc
2) Ureleroscopia con exlraccin del clculo. rapia.
3) Ncfrolitoloma endoscpica percutnea. 2) Realizar urografa intravenosa para intentar filiar la causa.
4) A lca lini zlcin de la orina por va oral . 3) Hidratar a la paciente baj o observacin rigurosa, y repetir ecografa a las 48
5) Admin ist racin de D-penicilamina. horas.
4) Colocar catter doble J o practicar nefrostomfa percutnca de forma inmediata
MIR 0304, 80; RC: 4 con cobertura antibitica.
5) laparotoma exploradora pJra objetivar causa, y re;lizar tratamiento antes de que
el cuadro est muy evolucionado.
Mujer de 50 aos, diabtica insulinodependiente, con infecciones urinarias y clicos
nefrticos de repeticin. Acude a Urgencias por dolor en fosa renal izquierda de cinco RC: 4

14
04.
TUMORES RENALES

Orll?ntacln Aspectos esenciales

MIR
El adenocarcinoma renal es un
tema "de moda en el examen El ms frecuente de los tumores slidos renales es el hipernefroma.
MIR. Cualquier aspecto de
este lema puede aparecer, El paciente caracterstico es un varn de mediana edad, obeso y fumador.
pero es fundamental reconocer
el sindrome de Stauffer, la trada clsica consiste en hematuria, dolor y masa en flanco, actualmente, lo ms habitual es que sea
que ha sido recientemente
incidentaloma (asintomtico). Si produce sntomas, el ms frecuente es la hematuria.
introducido y preguntado
varias veces consecutivas. Hay que sospechar tumor renal ante un varicocele izquierdo, de aparicin sbita y que no cede con el
decbito.

El hipernefroma puede producir multitud de sndromes paraneoplsicos. Esto puede complicar bastante el
diagnstico, de ah el sobrenombre de "tumor del internista".

la elevacin de las transaminasas sin afectacin heptica es tpica del hipernefroma (sndrome de Stauffer).

No se debe confundir un quiste simple con un hipernefroma. los criterios de quiste simple son: contorno
liso, contenido transnico y refuerzo posterior.

la primera prueba de imagen, ante la sospecha de hipernefroma, sera la ecografa.

El tratamiento fundamental del hipernefroma es la extirpacin quirrgica. la quimioterapia y radioterapia


tienen un papel muy secundario.

4.1 . Ca rci noma de c l u l a s ren a l es


(adenocarc i n oma ren a l, h i pernefroma)

Es e l tumor slido renal ms frecuente (90%) (Figura 8) (MIR 99-00, 1 77). Es un tumor fundamentalmente d e l a
edad adulta, con mayor incidencia entre los 40 y 60 aos, con predominio e n e l varn 2 : 1 a excepcin d e la
variedad cromfoba, t pi ca de las mujeres.

Entre los factores de riesgo que se han impl icado se encuentran el humo del tabaco, el cadmio y la obesidad.
Existen formas familiares que suelen ser mltiples y bilaterales, como en la enfermedad de Von Hippel -Li ndau y,
en menor medida, la esclerosis tuberosa (MIR 00-01 , 1 20). Se han identificado alteraciones cromosmicas que
implican al cromosoma 3 .

D RECUERDA Asimismo, existe una incidencia aumentada en el rin poliqustico,


en la enfermedad qustica adquirida de la insuficiencia renal crnica
la esclerosis tuberosa y la enferme
dad Von Hippel-lindau se asocian y en los riones malformados, como el rin "en herradura". Procede
tambin a otro tumor renal, el an de las clulas del tbulo contorneado proximal, y microscpicamente
III Preguntas giomiolipoma.
predominan las clulas claras sobre las granulares y fusiformes .

MIR 09-1 0, 99
- MIR 06-07, 102
. MIR 0506, 105 Presentacin
. MIR 04-05, 105
- MIR 01-02, 109
- MIR OO-Ol, 1 20
- MIR 99-00, 177
La trada clsica: hematuria, dolor y masa en el flanco ocurre nicamente en el 1 0% de los casos y, cuando se
- MIR 99-00F, 144
- MIR 98-99F, 146 presenta as, generalmente se trata de una enfermedad avanzada.

15
Manual (TO de Medicina y Ciruga, 8.a edicin

Figura 9. Ecografa de quistes renales simples

la realizacin de puncin-aspiracin con aguja fina (PAAF) de una


masa renal para su filiacin es una exploracin agresiva que, debido
Figura 8. Carcinoma de clulas renales
a su baja sensibilidad, no se justifica actualmente, excepto en casos
excepcionales.
El 30% presenta metstasis a distancia en el momento del diagnstico,
aunque contrariamente, cada vez son ms los hallazgos incidentales al la tomografa axial computarizada (fe) es el mejor mtodo aislado para
realizar ecografas abdominales de rutina por otra causa, alcanzando evaluar una masa renal, proporcionando informacin precisa sobre mets
en algunos estudios ms de la mitad de los casos diagnosticados. tasis ganglionares (80%) y afectacin de rganos adyacentes (Figura 1 0).

la anomala ms frecuente es la hematuria macroscpica o micros


cpica (60%). Otros hallazgos frecuentes son dolor (40%), prdida de
peso (30%), anemia (40%), masa en flanco (24%), HTA (20%), hiper
calcemia (6%), eritrocitosis (3%). El 20% de los pacientes presentan
como cuadro paraneoplsico alteracin de las enzimas hepticas sin
evidencia de metstasis (sndrome de Stauffer) (MIR 09- 1 0, 99; MIR 06-
07, 1 02 ; M I R 05-06, 1 05).

Ocasionalmente, el adenocarcinoma renal puede producir hormonas


productoras de sndromes clnicos segn la sustancia secretada. Entre
stas se encuentran pptidos PTH-like, prostaglandinas, prolactina, re
nina, gonadotropinas o corticoides. la invasin de la vena renal princi
pal en el lado izquierdo puede ocasionar la aparicin de un varicocele
de forma repentina, que no disminuye en decbito.

o RECUERDA
la produccin de pptidos puede aparecer en el hipernefroma, pero es
ms tpica de carcinomas epidermoides (pulmn, esfago, etc.).

"

"

Diag nstico
o
E
.=
La ecografa es la primera prueba complementaria que debe realizarse, La RM, aunque no se emplea de manera rutinaria en este tipo de pa
de forma que la identificacin ecogrfica de una lesin que cumple cri cientes, s se utiliza como mtodo de diagnstico bsico en sujetos en
terios de quiste simple (contorno liso, contenido transnico y refuerzo los que se sospecha afectacin lrombtica tumoral de la vena renal o
posterior) hace innecesarios mayores esfuerzos diagnsticos, pudiendo cava (MIR 04-05, 1 05) (Figura 1 1 ).
hacerse un seguimiento ecogrfico anual (MIR 99-00F, 1 44). De esta
forma se diagnostican l a mayora de las masas renales en la actualidad Aunque la urografa intravenosa (U IV) contina siendo la base del diag
(Figura 9). nstico por imagen en urologa, en el caso del adenocarcinoma renal

16
UrOIOga

proporciona pocos datos y de forma indirecta, como puede ser la dis Trata miento
torsin del sistema colector, su ocupacin o la anulacin funcional del
rin. En las placas tomogrficas de la UIV puede observarse la presen Una vez estudiado el tumor y descartada la presencia de metstasis,
cia de una masa o una alteracin del contorno renal. tanto viscerales como ganglionares, el tratamiento de eleccin es la
nefrectoma radical, incluyendo la fascia de Gerota y la glndula su
prarrenal (MIR 01 -02, 1 09).

En ocasiones especiales se puede plantear la ciruga conservadora o


parcial, como en los tumores bilaterales, en aquellos que aparecen so
bre rin nico, o sujetos con nefropatas mdicas, en los que la prdi
da de masa nefronal obligara a dilisis.

Adems de en estas indicaciones imperativas, actualmente es el nuevo


patrn de referencia en el tratamiento de tumores pequeos (menos de
4 cm), bien delimitados y sin afectacin de la grasa perirrenal. En estos
pacientes seleccionados parece que la supervivencia y la tasa de reci
divas locales son semejantes a las que se presentan en casos similares
tratados con nefrectoma radical.

La linfadenectoma regional no mejora la supervivencia y nicamente


tiene, por tanto, validez en la estadificacin, por lo que no se realiza
sistemticamente. ste se establece sobre la base de los hallazgos qui
rrgicos y anatomopatolgicos.
Figura 1 1 . R M de tumor renal con trombo en venas renal '1 cava

Otras formas de tratamiento carecen de eficacia. Tanto la quimiotera


La arteriografa renal, exploracin obligada hace aos, ha quedado re pia como la radioterapia ofrecen resultados pobres. En el caso de en
legada a los casos dudosos, riones nicos y otras situaciones en las fermedad metastsica, las opciones son mltiples, pero ninguna satis
que se plantea tratamiento quirrgico conservador. El patrn arteriogr factoria. Aunque se ha descrito la regresin de las lesiones metastsicas
fico caracterstico incluye neovascularizacin tumoral, lagos venosos, tras la nefrectoma, esto ocurre nicamente en u n 1 %, Y generalmente
fstulas arteriovenosas y vasos capsulares. de forma transitoria, por lo que no se justifica, salvo de forma paliativa
por otros motivos.
El estudio de extensin, si se sospechan metstasis, se completar rea
lizando radiografa de trax, analtica heptica completa y, en algunos La inmunoterapia con i nterferones, interleucina, l infocitos killer acti
casos dudosos, gammagrafa sea. vados, y ciertas combinaciones de quimioterapia con inmunoterapia,
son alternativas para la enfermedad metastsica, pero en ninguna de
El procedimiento diagnstico ante la presencia de masas renales se ellas se obtienen tasas de respuesta superiores al 1 5%. Actualmente
puede observar en la Figura 1 2. se utilizan de preferencia: antitirosincinasas en primera lnea, an
tiangiognicos en segunda lnea. Las situaciones que favorecen l a
respuesta al tratamiento i n m unoteraputico son: presencia d e me
tstasis pulmonares exclusivamente, buen estado general, y que se
MASA RENAL descubierta accidentalmente
haya realizado la nefrectoma a n tes del descubrimiento de las masas
I pul monares .
t
. Examen fsico
. Anlisis de orina
En la actualidad se i nvestiga sobre autovacunas elaboradas con linfoci
tos peritumorales que parecen ofrecer resultados alentadores.

Quiste complejo
o masa slida
Quiste simple
4.2. Otros tumores

Observar
Tumor d e Wilms (vase Seccin de Pediatra ) .
Tumores renales metastsicos. Pueden encontrarse metstasis en el
rin de tumores de pulmn (la ms frecuente), mama, mela nomas
Masa slida e infiltracin por linfoma.
o quiste complicado Angiomiolipoma
Tumores benignos:

/ Adenomas corticales: son los tumores ms frecuentes del adulto,


aunque indistinguibles clnicamente del adenocarcinoma, por lo
NEFRECTOMIA No complicado Complicado
que se tratan como tales. E l criterio clsico de tamao (3 cm)
RADICAL OBSERVACiN Nefrectoma simple
O PARCIAL Nefrectoma parcial para su diagnstico diferencial no es vlido en la actualidad.
Embolizacin Angiomiolipomas: se asocian a la esclerosis tuberosa en un 50%
(M I R 98-99F, 146). Compuestos de una proporcin variable de
Figura 12. Algoritmo diagnstico de las masas renales grasa, vasos y fibras musculares. Cuando son grandes (mayores

17
Manual ero de Medicina y Ciruga, B.a edicin

de 4 cm), pueden ocasionar u n sndrome de Wnderlich por guirlo del adenocarcinoma, pero en la mayora de los casos,
sangrado relroperitonea l . Cuando se asocian a esclerosis tubero ni stos ni la citologa o la biopsia, ofrecen garantas sufi
sa, suelen ser mltiples y bilaterales, por lo que deben tratarse de cientes de su benignidad, por 10 que tienden a ser tratados
forma conservadora. mediante nefrectoma.
Oncocitoma: considerado benigno, aunque en algunos se han Nefroma mesoblstico (hamarloma fetal): es el tumor benigno
detectado metstasis. Hay criterios radiolgicos para dislin- ms frecuente en recin nacidos y lactantes.

Casos clnicos representativos

Un hombre de 45 aos tiene un carcinoma de clulas renales extendido. los niveles Un paciente de 62 aos, con alteracin de la funcin renal y crisis de hematuria,
de GOl, fosfatasa alcalina, lOH y a-2 globulina son elevados y el tiempo de protom presenta una masa abdominal palpable en nanco derecho. Se le realiza una le, de
bina alargado. El hgado aparece difusamente agrandado, pero no existen defectos tectndose una masa de carcter slido de 8 cm de dimetro en rin derecho. En la
focales de infiltracin intraheptica. la explicacin etiolgica ms probable para anamnesis destaca que el paciente es fumador de 35 cigarrillos al da. Cul es, entre
estos hallazgos ser: los siguientes, el diagnstico de presuncin ms probable?

1 ) los efectos hcpdlolxicos de tumor. 1 ) Nefroblasloma.


2) Metsl,lSis hep.ltica. 2) Uposarcoma.
3) Amiloidosis. 3) Angiomiolipoma.
4) Trombosis tumorales que obstruyen la vena heptica. 4) Adenocarcinoma..
5) Hepatitis vrica aguda. 5) Carcinoma epidermoide.

MIR 05-06, 105; RC: 1 MIR 99-00, 1 77; RC: 4

18
05.
HIPERPLASIA y CARCINOMA
PROSTTICO

OrientacIn Aspectos esenciales

MIR
La hiperplasia prosttica benigna (HPB) suele afectar a la zona periuretral de la glndula. El cncer aparece
Tanto la hiperplasia
prosttica benigna como
[jJ en la zona perifrica.
el cncer de prstata, son
dos temas fundamentales. La hiperplasia prosttica benigna no guarda relacin con el cncer.
Probablemente el cncer sea
ms importante, sobre todo Tanto la HPB como el cncer tienen relacin con las hormonas sexuales, y suelen aparecer en varones
en lo referente al tratamiento. ancianos.
Hay que aprenderse muy bien
el resumen de la Tabla 1 1; El tratamiento mdico de la HPB consiste en a-bloqueantes (relajan la musculatura uretral y del cuello
aporta muchas preguntas vesical), inhbidores de la 5 a-reductasa (disminuye el tamao glandular) y (toterapa. Esta ltima no ha
acertadas a cambio de poco demostrado utilidad con parmetros objetivos.
esfuerzo.
El tratamiento definitivo de la HPB es la ciruga, que puede consistir en reseccin transuretral o en ciruga
abierta, dependiendo del tamao prosttico.

El cncer de prstata es casi siempre un adenocarcinoma, con gran frecuencia multifocal.

El cncer de prstata cada vez se diagnostica con ms frecuencia en fase asintomtica. Cuando presenta
clnica, puede consistir en sntomas urinarios similares a la HPB.
El PSA elevado no es diagnstico de cncer de prstata. Puede corresponder a una HPB. El diagnstico defi
nitivo de cncer prosttico precisa una biopsia.

Las metstasis lumbares son tpicas del cncer de prstata, pudiendo producir compresin medular.

El tacto rectal revela una prstata ptrea e irregular en el cncer de prstata. Sin embargo, al principio puede
no ser palpable, ni visible en la ecografa (T1).

La principal complicacin quirrgica del cncer de prstata es la impotencia.

Ante un sndrome de compresin medular por cncer de prstata, nunca se deben emplear anlogos de la
LHRH nicamente. Siempre deben asociarse antiandrgenos.

En el cncer de prstata, la indicacin ms clara de prostatectoma radical es el estadio T2a.

El tratamiento fundamental del cncer de prstata diseminado es la hormonoterapia.

5 . 1 . H i perplasia prosttica ben i g na

La hiperplasia prosttica benigna (HPB) afecta en mayor o menor grado a la gran mayora de los varones a partir
de la quinta dcada de la vida, alcanzando el 80-95% de la poblacin masculina de 80 aos.

La prstata se divide clsicamente en cinco lbulos (anterior, medio, posterior y dos laterales); aunque stos ni
camente se encuentran como tales en la edad fetal. En el adulto se puede interpretar la anatoma de la prstata
dividida en dos partes: una zona perifrica, donde se origina principalmente el carcinoma, y una zona periure
iIl Preguntas tral o transicional, de la que procede la HPB (Figura 1 3) .

MIR 08-09, 106


La HPB est compuesta de una proliferacin variable de elementos glandulares, musculares y del estroma, que en
- MIR 06-07, 1 03, 233
- MIR 05-06, 106 su crecimiento comprimen la prstata perifrica, formando la l l amada cpsula quirrgica. Su etiopatogenia no est
- MIR 04-05, 1 06, 225 clara; aunque el estmulo andrognico a travs de su forma activa, la dihidrotestosterona, es fundamental, su papel
- MIR 03-04, 91
- MIR 02-03, 188 exacto no ha sido determinado. Las teoras ms recientes abogan por un desequil ibrio hormonal de estrgenos/an
- MIR 01 -02, 1 04, l OS, 1 07 drgenos, o fX'r la existencia de factores de crecimiento prostticos con un papel permisivo del ambiente hormonal.
- MIR aQ-Q1 , 1 1 9
- MIR 98-99, 1 3 7
- M I R 98-99F, 147 No existe evidencia de asociacin entre HPB y carcinoma prosttico.

19
Manual eTO de Medicina y Ciruga, B.a edicin

Fase clnica (Figura 1 5). La elongacin de las fibras musculares por


Estroma fibromuscular anterior
encima de un lmite condiciona prdida de capacidad contrctil. En

\?7
este momento aparece retraso del inicio de la miccin, disminucin
del calibre y de la fuerza del chorro micdonal y alargamiento del
vaciado (10 que en conjunto se denomina sndrome prosttico), El va
ciado suele ser incompleto, dando lugar a un residuo postmiccional.

--=,- Zona perifrica

zona central

Glndulas
Lbulo anterior suburetrales
Conductos
eyaculadores

Lbulo
lateral

Lbulo posterior

Figura 13 . Anatomra de la prstata


Urografa lntraveno mostrando Impronta prosttica en vejiga (vejiga en montera")
con urteres en anzuelo

Diagnstico Figura 1 S. Hiperplasia prosttica en fase clnica

Fase de descompensacin (Figura 1 6). Se produce un vencimiento


El crecimiento prosttico generalmente se produce hacia la uretra, del detrusor vesical, que es incapaz de vencer la presin uretral, au
ocasionando obstruccin de sta y dificultando el vaciamiento vesical mentando la sintomatologa anterior y pudiendo presentarse reten
(Figura 1 4). Esto no se manifiesta inmediatamente, sino que, general cin urinaria. Ocasionalmente puede aparecer dilatacin ureteral
mente, el proceso pasa por una serie de etapas que incluyen una fase bi lateral con deterioro de la funcin renal. Esto se debe a uropata
de compensacin, una fase clnica y una de descompensacin. obstructiva infravesical con prdida del mecanismo antirreflujo.

8
"
19
e"

E
o
e
'2

u
>
.
,

c.
"
"-
I Figura 14. Ecografa de hiperplasia prosttica con crecimiento
del 16bulo medio intravesical Situacin del mismo paciente un ao despus de la anterior: hidronefrosis grave.
De la vejiga (no se observa) se evacuaron 3500 ce de orina

Fase de compensacin. El crecimiento prosttico ocasiona un au Figura 16. Hiperplasia prosttica en fase de descompensaci6n

mento de la presin uretral durante el vaciado que es compensado


por una mayor actividad contrctil del detrusor que se hipertrofia, Pueden producirse tambin otro tipo de sntomas denominados "irrita
encontrando presiones vesicales ms elevadas. En esta fase, la clni tivos", que son debidos a la alteracin funcional vesical, y cuya reso
ca puede ser mnima o inexistente. lucin es ms difcil tras la desaparicin de la obstruccin (MIR 01 -02,

20
u rolo 9ia a
1 04). Entre estos sntomas se incluyen polaquiuria, tenesmo, nicturia y doxazosina, terazosina, tamsulosina, etc.) que relajan la musculatura
urgencia mlccional. La HPB es la causa ms frecuente de obstruccin del cuello vesical y uretra. Hasta ahora, estos frmacos se han estado
del tracto urinario inferior en el varn. utilizando en forma de escalera teraputica, pero la aparicin del es
tudio COMBAT parece indicar que en pacientes con sintomatologa a
En la evaluacin del sndrome prosttico, el tacto rectal contina sien partir de moderada, y con volmenes prostticos por encima de 30-40
do la exploracin fundamental, sobre todo para diferenciarlo del car ce se debe realizar terapia combinada de inicio.
cinoma, ya que no es infrecuente que ambas entidades coexistan. La
clnica es lo ms importante para valorar la indicacin de tratamiento
de la HPB, ya que no existe correlacin entre el tamao prosttico y el
o RECUERDA
La finasterida tambin es til para la alopecia androgni ca, donde se
grado de obstruccin (MIR 08-09, 1 06). Cualquier zona sospechosa al emplea en dosis mucho menores.
tacto debe ser biopsiada.

La medicin del flujo mximo miccional es tambin importante, conside Como inconvenientes principales de los inhibidores de la 5 a-reductasa,
rndose normal cuando es mayor de 1 5 mi/s y claramente patolgico si se encuentran: i mpotencia, reduccin del PSA en torno al 50% (difi
es menor de 1 0 mi/s. El estudio puede completarse con una ecografa que cultando el diagnstico del carcinoma, si lo hubiese) y que tarda una
permita evaluar si existe afectacin del tracto urinario superior, residuo media de cuatro meses en hacer efecto.
postmiccional, litiasis vesical u otra patologa asociada. El uso del PSA en
la HPB nicamente est indicado para descartar la presencia de carcino De los a-bloqueantes, el inconveniente principal es la hipotensin.
ma en la prstata, ya que no sirve para diagnosticar HPB, aunque recien
temente ha demostrado ser el mejor predictor de la historia natural de la En cuanto a las indicaciones de tratamiento quirrgico, globalmente,
enfermedad. Es decir, que mayores niveles de PSA en HPB diagnosticada slo un 1 0% de los pacientes prostticos precisar ciruga. La inten
probablemente se correlacionarn con mayores volmenes prostticos y sidad de las manifestaciones clnicas subjetivas y la mala respuesta al
con ms posibilidades de complicacin derivadas de la HPB. tratamiento mdico pueden constituir la indicacin para la interven
cin. Entre las causas "objetivas" que suponen indicacin absoluta de
tratamiento quirrgico se encuentran (MIR 01 -02, 1 05):
Trata miento Retencin urinaria reiterada.
Hidronefrosis retrgrada (lesin del parnquima renal por obstruc
cin infravesical).
Dentro de las posibilidades teraputicas, la ciruga contina siendo el Infeccin urinaria de repeticin.
nico tratamiento definitivo para la HPB. tsta puede ser endoscpica Litiasis vesical.
(RTUp: reseccin transuretral prosttica) o abierta (adenomectoma pros Hematuria de repeticin.
ttica) (Figura 1 7), dependiendo del tamao del adenoma. En el 1 0% de
las piezas obtenidas se encontrarn focos de adenocarcinoma incidental.

5.2. Carci noma p rosttico

El adenocarcinoma prosttico es el tumor maligno ms frecuente del


aparato genitourinario masculino y el segundo en frecuencia general,
despus del pulmonar. Sin embargo, si se incluyesen los carcinomas
incidentales y los encontrados en autopsia, supera al pulmonar en pre
valencia (MIR 06-07, 1 03).

La hormonodependencia del cncer prosttico parece indicar el papel


de los andrgenos en su etiologa o patogenia. La relacin de factores
genticos, ambientales o infecciosos no ha quedado suficientemente
establecida.

El 95% de los carcinomas prostticos son adenocarcinomas originados


en la zona perifrica de la prstata. Los carcinomas ductales se originan
en los conductos prostticos en lugar de los acinos, e histolgicamente
Figura 17. Adenomectomia prost ti ca
pueden corresponder a carcinomas transicionales, escamosos, endome
trioides o mixtos. Ms raros son los carcinosarcomas (menos del 1 %).
Se debe tener en cuenta que en la ciruga de la HPB no se extirpa la
cpsula quirrgica, que est constituida por las glndulas prostticas El adenocarcinoma prosttico, con frecuencia, es multifocal y presenta
perifricas comprimidas por el adenoma, y es el principal origen del poblaciones en distinto grado de diferenciacin. En esta heterogenei
carcinoma prosttico, por lo que la intervencin quirrgica no protege dad se basa la clasificacin de Gleason, que asigna una puntuacin de
del desarrollo de este proceso. 1 a 5, segn el patrn histolgico de cada una de las dos poblaciones
ms representativas de la masa, sumando ambas puntuaciones para
Los tratamientos no quirrgicos incluyen una variedad de fitoterapias, obtener un resultado final de 2 a 1 0. Esta escala de Gleason se corres
poco efectivas si se valoran con parmetros objetivos, inhibidores de la ponde con el pronstico de la enfermedad, independientemente del
5 a-reductasa (finasterida, dutasterida) (MIR 04-05, 225) que reducen el estadio (MIR 06-07, 233). Para la estadificacin se emplea principal
tamao prosttico, antagonistas a-adrenrgicos (alfuzosina, prazosn, mente la clasificacin TNM (Tabla 1 0 y Figura 1 8) (MIR 98-99F, 1 4 7).

21
Manual eTO de Medicina y Ci rug a, s.a edicin

Cl nica

. T1: tumor naparente cI(nicamente (no pal pable ni visible por tcnicas de
i magen) El carcinoma prosttico es una enfermedad ms frecuente en ancia
T1 a: hallado incidentalmente. Afectacin menor del 5% del tej ido resecado nos, y la mayora de ellos se diagnostica por encima de los 60 aos.
T l b: hallado incidentalmente. Afectacin mayor del 5% del tejido resecado
Clnicamente puede producir sntomas obstructivos del tracto urina
Tlc: Tumor identificado por p u ncin-biopsia por aumento del PSA

T2: tumor confinado a la prstata (in cl uye la inva sin de la cpsula prosttica rio inferior superponibles a los de la HPB. A stos puede aadi rse la
sin exteriorizacin del tumor hacia el tejido adiposo periprosttico) hematuria. El 25% de los pacienles que refieren retencin urinaria
T2a: menos del 50% de u n lbulo aguda presentan u n carcinoma prosttico. Aproximadamente u n 25%
- T2b: ms del 50% de un lbulo
- T2c: dos l bulos
de los pacientes presentan metstasis en el momento del diagnstico;
T3: extensin del tumor por fuera de la cpsula stas pueden producir man ifestaciones como dolor seo, compresin
- T3a: extensin transcapsular (sea unilateral o bilateral) medular, mieloptisis o coagulopata. Afortunadamente, estos casos
- T3b: invasin de la(s) veslc u l a (s) seminal{es)
se encuentran en claro descenso gracias a la i ncorporacin del PSA
T4: tumor fijo o invade rganos adyacentes distintos a las vesculas
(prostate-specific a n tigen -a n tgen o prosttico especfico), facilitando
sem inales (cuello vesical, esfnter externo, recto, msculo elevador
o pared el diagnstico de la enfermedad en estadios tempranos y comnmen
te asintomticos.
NX: no se pueden estudiar los ganglios regionales
NO: no metstasis ganglionares
N 1: metstasis a Diagnstico

MO: no metstasis
M l : metstasis a distancia Tacto rectal
Ml a: ganglios linfticos no regionales
Mlb: h ueso
M 1 c: otras local izacion es Contina siendo el mtodo fundamental de cribado. Son accesibles al
tacto rectal todos los estadios excepto el T1, que por definicin es un
Tabla 1 0. Estadificacin del carcinoma de prstata
hallazgo. Caractersticamente, el carcinoma es duro, nodular e irre

T: tumor primario cUnlca TN


na

Tla Tlb n
TX NO puede evaluar el tumor
TO No existen signos de tumor primario

na nb

Tlc nb

<5% >5%

T1 Tumor no evidente dlnic.amente, no palpable T2 Tumor limitado a la prstata o a la T3 Tumor que se extiende a travs de la
ni visible mediante tcnicas de imagen: capsula, sin sobrepasarla: capsula prosttica:
na Extensin menor o igual a1 5% del tejido T2a Menos del 50% de un lbulo . TIa Extensin extracapsular (unilateral
resecado T2b Ms del 50% de un lbulo o bilateral)
. Tl b Extensin mayor del 5% del tejido T2c Dos lbulos . TIb Tumor invade la veslcula seminal
resecado
. TlcTumor identificado mediante puncin
bipsica (consecuenc.ia de un PSA elevado)
N: ganglios linfticos regionales

T4 Tumor fijo o que invade estructuras adyacentes diferentes a las veslculas seminales NX No se pueden evaluar los ganglios linfticos regionales
NO No hay metstasis ganglionares regionales
Nl Metstasis en ganglios linfticos regionales

Figura 18. Estadificacin del adenoca rd noma de prstata

22
Ur0109a .a
guiar. En general, se aconseja un tacto rectal y un PSA anual a todos tivo se ha i ntentado aumentar su especificidad para cncer con otros
los varones por encima de 50 aos aunque, de momento, la OMS no parmetros (densidad de PSA, ndice PSNedad, velocidad de cambio
aconseja la realizacin de cribado poblacional sistemtico. del PSA, PSA libre), aunque an no ha quedado establecida la ventaja
de stos sobre el PSA aislado.

Marcadores tumorales Si el PSA es menor de 4 ng/ml, es poco probable que se encuentre un


cncer de prstata. Si es mayor de 1 0, las probabi lidades aumentan, lo
Se dispone fundamentalmente de dos marcadores tumorales. La fosfa que aconsejara una biopsia de prstata ecodirigida. S i est entre 4 y
tasa cida prosttica (FAP) se emplea en clnica desde hace dcadas, 1 0, se pueden utilizar los parmetros antes mencionados para valorar
es un marcador especfico, pero su elevacin suele indicar extensin la necesidad de biopsia (MIR 0 1 -02, 1 07).
extraprosttica, por lo que no resulta til en el diagnstico precoz. El
antgeno prosttico especfico (PSA) es realmente un marcador de te
jido prosttico cuyos niveles suelen encontrarse ms elevados en el Pruebas de imagen
cncer, pero es inespecfico y tambin estn elevados a consecuencia
de patologa benigna (infecciones, sondajes, HPB, etc.). Por este mo La ecograra transrectal (ETR) (Figura 1 9) es el mtodo de imagen ms
til para la esladificacin local, pudiendo ofrecer informacin impor
tante sobre la afectacin capsular, de vesculas seminales, cuello vesi
cal o recto. Aunque no existe un patrn caracterstico, suele aparecer
como ndulos hipoecognicos. La ecografa transrectal ofrece, ade
ms, la posibilidad de dirigir la biopsia hacia las zonas sospechosas.

La ecografa abdominal no tiene gran valor en la deteccin del car


cinoma prosttico. La TC y la RM tienen su principal papel en la es
tadificacin ganglionar y la valoracin de metstasis a distancia. Las
primeras metstasis deben buscarse a nivel de los ganglios l infticos de
las cadenas obturatrices e ilacas.

Gammagrafa sea

Se utiliza para la deteccin de metstasis seas, tiene mayor sensibi


lidad que la radiologa convencional (Figura 20), y debe realizarse en
todo paciente en quien se sospeche metstasis (Gleason > 8, PSA > 20).
Antes de plantearse el tratamiento curativo, en ciertos pacientes con al
tas probabil idades de encontrarse el cncer extendido, se debe efectuar
una gammagrafa previa para confirmar la no existencia de metstasis
seas o una TC para descartar metstasis ganglionares.

I) ETR corte transversal; {b} ETR corte longitudinal; (el Adenocarclnoma,


ndulo hlpoecojco en lbulo derecho

Figura 19. Ecografra transrectal (ETR) de adenocarcinoma prosttico Figura 20. Radiografa de columna. Metstasis osteoblsticas

23
Manual CTO de Medicina y Ciruga, 8," edicin

D RECUERDA Agonistas LHRH. Aunque inicialmente ocasionan un aumento


de los niveles de testosterona, posteriormente suprimen la se
las metstasis del cncer de prstata son osteoblsticas, es decir, for
man hueso (no en la gammagrafa, sino en la rad iografa) . crecin de lH y de andrgenos. la elevacin transitoria de los
andrgenos puede empeorar el cuadro clnico, principalmente
si existe compromiso medular por metstasis seas. Esta eleva
cin rflare-up) se debe suprimir mediante la administracin de
Biop sia prosttica antiandrgenos, previamente a la introduccin de inhibidor de
la LHRH (MIR 00-0 1 , 1 1 9).
Debe realizarse para la confirmacin del diagnstico. Puede efectuarse Antiandrgenos (bicalutamida, flutamida, acetato de ciprotero.
va transrectal o transperineal, guiada por el tacto rectal o bien guiada na). Compiten con el receptor andrognico. Suelen utilizarse con
por la ecografa transrectal, lo que aade efectividad a la prueba. la con agonistas de la lHRH. El acetato de ciproterona, adems de
realizacin de la biopsia est indicada siempre que exista una anoma actuar como antiandrgeno, tiene un efecto progestgeno, por lo
la del tacto rectal, elevacin de los marcadores tumorales o alteracin que acta a nivel central, disminuyendo los pulsos de lH.
en las pruebas de imagen. la puncin-aspiracin con aguja fina (PAAF) Quimioterapia. No es muy efectiva en el adenocarcinoma pros
es una alternativa con menores complicaciones, pero con el inconve ttico. Se han realizado tratamientos con frmacos, que son una
niente de que no puede evaluar el grado histolgico (Gleason). mezcla de un estrgeno y una mostaza n itrogenada (fosfato de
estramustina), pero los estudios son contradictorios y las res

D RECUERDA puestas pobres. Existen estudios prometedores con el empleo de


docetaxel en pacientes con tumores hormonorresistentes.
Son indicaciones de b iops ia prosttica el tacto rectal sospechoso, la
presencia de un ndulo ecogrfico y un PSA > 4 (variable la cifra segn
criterios).

Tratamiento por estadios (MIR 04-05, 1 06)

Estadio Tla. Tienen una mortalidad por la enfermedad del 2% a los


Trata miento 1 0 aos, por lo que no precisan tratamiento, salvo quizs los pacien
tes jvenes (menores de 60 aos) con una elevada esperanza de vida.
Estadio Tlb-Tlc. Alcanzan una mortalidad del 80% dejados a su
Opciones teraputicas evolucin natural. Por ello est indicada la prostatectoma radical,
la radioterapia externa o braquiterapia, en sujetos con esperanza de
Prostatectoma radical. los pacientes candidatos deben ser individuos vida superior a 1 0 aos (MIR 02-03, 1 88) .
con una esperanza de vida superior a 1 0 aos. Como complicaciones, Estadio T2a. Es la indicacin ms clara de prostatectoma radical. la
se puede encontrar incontinencia (2-57%), estenosis anastomtica radioterapia o braquiterapia se reservara para pacientes de riesgo
(1 0%), impotencia (50%) o incluso la muerte 5%). En l neas gene quirrgico elevado o que no aceptan efectos secundarios atribuibles
rales, suele ir acompaada de linfadenectoma leo-obturatriz. a la ciruga.
Radioterapia. Como tratamiento curativo los resultados en estadios
I Estadio T2b y T2c. Un 40% demuestra ser en realidad estadio 3, tras
localizados se acercan a los de la ciruga. la diarrea crnica, la el anlisis de la pieza qu irrgica de prostatectoma radical (infraes
proctitis, la cistitis rdica y las fstulas urinarias son complicaciones tadificacin). la radioterapia externa o braquiterapia tambin puede
del tratamiento, as como la incontinencia y la impotencia a partir ser til en pacientes de alto riesgo quirrgico.
de los dos aos de tratamiento. Estadio T3a. la i ndicacin quirrgica es dudosa, as como la radio
Se ha empleado tambin radioterapia intersticial (braquiterapia) con terapia local, por lo que solamente se propondra a sujetos jvenes,
implantacin de yodo-1 23 (1- 1 23), oro-1 98 (Au-1 98), paladio e iri aun a costa de obtener malos resultados. Generalmente son tratados
dio. Su indicacin queda limitada a tumores pequeos de estadio T1 como el grupo siguiente.
o T2, y sus resultados son similares a los de la ciruga. En caso de Estadio T3b, T4, N+, M+. Varn aoso con mal estado general. El
compresin medular o dolor por metstasis seas, la radioterapia so tratamiento hormonal es la opcin indicada. Puede ser preciso el
bre la metstasis puede conseguir el control local de la enfermedad. uso de radioterapia paliativa sobre la metstasis en caso de dolor
Hormonoterapia. El adenocarcinoma prosttico est compuesto por (MIR 03-04, 9 1 ; MIR 98-99, 1 3 7).
una poblacin heterognea de clulas andrgeno-dependientes y
andrgeno-independientes. La supresin hormonal frena el creci
miento de las primeras, pero no afecta a las a ndrgeno-indepen Recidiva tumoral posterior a tratamiento con intencin curativa
dientes. Se puede conseguir disminuir los niveles de andrgenos
circulares por distintos mtodos: Tras la realizacin de prostatectoma radical, los pacientes son monito
Castracin quirrgica. Es el mtodo aislado ms eficiente, con la rizados generalmente con peticiones de PSA
ventaja de que elimina la necesidad de medicacin permanente.
Por su rapidez en el efecto supresor hormonal, tambin est in Cuando las cifras de PSA tras prostatectoma radical son superiores a
dicada en las compresiones medulares por metstasis. 0,4 ng/ml, se considera recidiva bioqumica y debe hacer sospechar la
Estrgenos (dietilestilbestrol). Inhibe la secrecin de LH. Actua existencia de metstasis a distancia, o bien la existencia de recidiva a
lemente este mtodo se ha abandonado debido al alto riesgo nivel local (MIR 05-06, 1 06)_
cardiovascular que conlleva.
Progestgenos. Inhiben la secrecin de lH y actan como an Tras la realizacin de radioterapia como tratamiento de cncer de
tiandrgenos, al unirse a los receptores de la dihidrotestosterona. prstata localizado, los descensos de PSA van siendo paulatinos (a di
Es preciso aadir estrgenos para evitar el fenmeno de escape, ferencia de la prostatectoma radical) hasta conseguir un valor nadir,
que se produce tras varios meses de tratamiento. No son de uso que es el valor mnimo alcanzado tras el tratamiento y que se con
habitual. siderar referencia para el seguimiento posterior. Existen diferentes

24
UrO,09a a
criterios para considerar el diagnstico de recidiva bioqumica tras I ADENOCARCINOMA
"PB
tratamiento con radioterapia: cuando se constata la existencia de tres PROSTATICO

elevaciones sucesivas a partir del valor nadir, cuando se evidencian Locotlucln Zona translcional Perifrica
n iveles nadir+2 (criterio de la ASCO), o con valores nadir+3 (criterio
Fases: 25% slndrome prosttico
de Philadelphial. Compensacin 25% retencin aguda
Olnlca
CHnica 25% metstasis
Descompensacln

Tratamiento de urgencia Tacto rectal


Ecografla transrectal (estadificacin local)
Gammag rafla osea (metstasis seas)
OIognOstlco
La compresin medular por el cncer prosttico no tratado puede ser la PSA (muy sensible, poco especifico). Descarta cncer prosttico,
d_.1
forma de presentacin y constituye una urgencia importante. El objetivo pero no diagnostica HPB
Fosfatasa cida (muy especifica, poco sensible)
del tratamiento debe ser la supresin andrognica rpida o la descom Biopsia (confirmacin)
presin medular mediante laminectoma quirrgica o radioterapia. Se
Fitoterapia Localizado: prostatectomla radical
pueden disminuir los niveles de andrgenos mediante castracin quirr Frmacos: finasterida, ms Ilnfadenectomla bilateral.
gica urgente, ketoconazol en altas dosis o dietilestilbestrol i ntravenoso. Tratamiento n-bloqueantes radioterapia
Clrugla: adenomectomla: Avanzado: castracin: quirrgica
endoscplca o abierta (eleccin), farmacolgica
A continuacin se expone una tabla que resume tanto las caractersti
cas de la hiperplasia prosttica benigna (HPB) como las del adenocar Tabla 1 1 . Tabla-resumen de las caracterfstlcas de la HPB
y del adenoca rcJnoma prosttico
cinoma prosttico (Tabla 1 1 l.

Casos clnicos representativos

Paciente de 66 aos, intervenido de prostatecloma radical, hace 3 aos por ade Un paciente de 67 aos acude a Urgencias por presentar en los ltimos das de
nocarcinoma de prstata Cleason 8 (pT2b NOMO). Presenta, en el momento actual, bilidad progresiva de miembros inferiores, dificultad miccional e incontinencia
una cifra de PSA srico de 1 2 nglml. Seale cul de las siguientes afirmaciones le fecal. En la exploracin fsica destaca cierta hipotona anal, con una prstata muy
parece correcta: sugerente de malignidad al lacto, y debilidad de extremidades, conservando la sen
sibilidad tctil. Con el probable diagnstico de carcinoma de prstata melastsico,
1) La supervivencia media en el momento actual es menor de 1 ao. cul de las siguientes opciones considera MENOS indicada para el tratamiento
2) La cifra de PSA est en rango normal ya que existen otras fuentes de produccin de urgencia?
del mismo.
3) El paciente puede tener una recidiva local o bien melstasis a distancia. 1 ) Estrgenos intravenosos.
4) La utilizacin de bloqueo hormonal en este caso no es una opcin de tratamiento 2) Anlogos LHRH.
posible. 3) Ketoconazol (altas dosis) .
5) En caso de tratarse de una recidiva local, estara indicado realizar ciruga de res. 4) Radioterapia.
cate para extirpar dicha masa. 5) Orquiectoma bilateral.

MIR 05-06, 106; RC: 3 MIR 00-01 , 1 1 9; RC: 2

Hombre de 77 aos que refiere clnica de prostatismo de aos de evolucin, y que A la consulta acude un paciente de 54 aos con molestias a la miccin. Refiere
presenta elevacin del PSA (antgeno prosttico especifico) (89 ng/ml) y dolor en disminucin del c,",orro, dificultad para el inicio, goteo postmiccional, sensacin de
columna lumbar desde hace 2 meses. Al tado rectal, la prstata est aumentada de tenesmo y nicturia de tres veces. Presenta cultivos negativos y PSA de 2,1. En la
tamao, de consistencia dura en ambos lbulos, superficie nodular y lmites mal defi ecograffa abdominal se objetiva una glndula prosttica de 43 cc. Al tacto reclal
nidos. Tras realizarle una ecografa transrectal con biopsias prostticas ecodirigidas, no se palpan ndulos sospechosos. En el IPSS obtiene una puntuacin que permite
es diagnosticado de un adenocarcinoma de prstata pobremente diferenciado, que clasificar su sintomatologa de moderada-grave. Su actitud deber ser:
afecta a ambos lbulos y que infiltra las vesculas seminales. La gammagrafa sea
confirma la presencia de metslasis en columna lumbar. Qu tratamiento de los 1) Debido a su edad, el primer paso ser iniciar tratamiento con Htoterapia.
siguientes aconsejara en primer lugad 2) Debido a la gravedad de los sntomas se debe plantear ciruga de entrada.
3) Se debe iniciar tratamiento con a-bloqueantes.
1 ) Prostalectoma radical. 4) La mejor opcin ser i nicia r tratamiento combi nado con a-bloqueantes+inhibidores
2) Quimioterapia intensiva. de la 5 a-reductasa.
3) Hormonoterapia. 5) Se debe iniciar tratamiento con inhibidores de la 5 a-recluctasa.
4) Radioterapia pelviana externa.
5) Braquiterapia prosttica. Re: 4

MIR 03-04, 9 1 ; RC: 3

25
06.
CARCINOMAS
DEL TRACTO URINARIO

Onentacln Aspectos esenciales

MIR
Las preguntas sobre este
tema suelen ser senciHas
GJ El carcinoma vesical ms frecuente es el urotelial, siendo el tabaco el principal factor de riesgo.
y repetitivas, aunque
ltimamente ha aparecido (Il El carcinoma escamoso se relaciona con la esquistosomiasis (5. haemafobium).
como nuevo concepto el
carcinoma in silu. El estudio (I) El adenocarcinoma vesical se relaciona con el antecedente de exLrofia vesical.
del Desglose es especialmente
importante, pero se debe (i) El carcinoma papilar superficial y el carcinoma in situ (OS) son muy recurrentes.
tener en cuenta que la
tendencia parece orientada
a preguntarse cada vez ms.
(I) Clnica ms frecuente del carcinoma urotelial: hematuria, ms tpico con cogulos.
Es fundamental la parte de
tratamiento. m Cuando se trata de un carcinoma in situ: sntomas rritativos (polaquiuria, disuria, tenesmo, etc).

m El mejor mtodo para la estadificacin local es la reseccin transuretral.

ill Prueba ms sensible para el diagnstico de OS: citologa urinaria.

(2J Conducta ante un ClS: tratar con bacilo Calmette-Guerin (BCG ) y revisiones (cistoscopia y citologas).

G:Q) Actitud ante un tumor superficial: reseccin transuretral. Posteriormente, revisiones (CiSloscopia y citologas).

G:IJ Actuacin ante un tumor infiltrante (afectacin capa muscular): cistectoma.

6 . 1 . Carci n o m a vesical
E l carcinoma vesical es l a segunda neoplasia urolgica e n frecuencia. Aparece ms frecuentemente en varones
(2-3 : 1 ) y ms en poblacin blanca que negra. Su edad de mxima incidencia se sita entre los 60 y 70 aos. De
el los, el 90% son carcinomas transicionales, el 8% escamosos y el resto adenocarcinomas. El epitelio uroteJial
recubre el tracto urinario desde las papilas caliciales hasta la uretra prosttica, ambas incl usive. En cualqu iera
de estos niveles pueden desarrollarse los tumores uroteliales, correspondiendo la mayora a la vejiga (ms del
90%) y, ms raramente, al tracto urinario superior (5%) o la uretra (1 %).

Entre los factores etiolgicos (Tabla 1 2), se implican las ami nas aromticas, presentes en las industrias textiles,
qumicas y del caucho. El humo del tabaco es el principal factor de riesgo (50-60% aparecen en fumadores),
aumentando el riesgo a mayor consumo. Tambin pueden jugar un papel importante los edulcorantes artificiales
(sacarina, ciclamato), la ciclofosfamida, los acetiladores lentos (mayor riesgo) y muchas otras posibles etiologas.
La infestacin por Schistosoma haematobium aumenta la incidencia de carcinoma escamoso vesical, as como
la presencia de infeccin crnica o catter vesical permanente.

Aminas aromtlcas {2-naftllaminal; tabaco, industria


eL Preguntas textil, industria del caucho, colorantes
Fenacetinas crnicas Translclonales (90%)
- MIR 07-08, 102 Sacarina, ciclamato Mejor pronstico
- MIR 06-07, 96 Ciclofosfamida (acrolelna)
- MIR 05-06, 103 Tabaco: ortofenoles, trlptfano
- MIR 04-05, 259
- MIR 03-04, 82 Schfsrosoma haematobium
Escamosos (8%)
- MIR 02-03, 1 79 litiasis, infecciones, catteres
- MIR 00-01 , 1 1 8
Cistitis glandular
- MIR oo-O I F, 1 43, 144 Adenocarclnomas (2%)
Extrofla vesical
- MIR 98-99, 136
- M I R 98-99F, 1 4 5
Tabla 12. Factores etiolgicos de los carcinomas del tracto urinario

26
UrOI09a a
El adenocarcinoma primario vesical es un tumor raro, aunque es el que
se ha visto asociado a la extrofia vesical con mayor frecuencia.

o RECUERDA
No hay que confundir Schisrosoma haemarobium con SchislOsoma
mansoni, que prod uce hipertensin portal.

Histologa e historia natural

Haciendo referencia al carcinoma de clulas transicionales, hay que di


ferenciar tres formas de la enfermedad con comportamiento, pronstico
y tratamiento completamente distinto (Tabla 1 3 y Figura 2 1 ). El 70% de
los tumores vesicales se presentan como tumores papilares de crecimien
to principalmente endocavitario y frente de invasin nico. Un 1 0% son
1. Mucosa ----;. Ta,Tis 4. Grasa -----+J T3
slidos, con invasin tentacular en profundidad y extensin linftica y vas
2.5ubmucosa J T1 5. rganos vecinos _ T4
cular temprana. El 20% restante son formas mixtas. El primer grupo suele
3. Muscular ----+J 12
corresponder a tumores superficiales de bajo grado histolgico, mientras
que los slidos, con mayor frecuencia, son tumores infiltrantes de grado
histolgico ms elevado. la principal caracterstica de los tumores papi Figura 2 1 . Esquema de la estadificacin del tumor vesical
lares superficiales es la recurrencia, que ocurre en un 50-75%, segn el
grado y estadio. El 25% recurrirn y progresarn en grado y estadio, y ni
camente e1 1 5% acabar desarrollando un tumor infiltrante o metastsico. Se puede encontrar en el mbito vesical disti ntas lesiones benignas que
no se asocian con el desarrollo de cncer: los nidos de von Brunn, la
la mayora de los tumores infi ltrantes se encuentran confinados a la cistitis qustica y glandular origi nados en procesos inflamatorios o rrila
vejiga en el momento del d iagnstico, y slo un 20-25% presentan tivos crni cos, y que probablemente sean distintas manifestaciones de
extensin ganglionar o metastsica. El 50% desarrollarn metstasis a un mismo proceso, aunque pueden plantear el diagnstico diferencial;
distancia, a pesar del tratamiento (MIR 04-05, 259). (ocasionalmente se han descrito adenocarcinomas vesicales asociados
a la cistitis glandular). Olras lesiones benignas seran el adenoma nefro
El tercer grupo que merece mencin aparle es el carcinoma in situ. A gnico, el plipo simple, el papiloma i nvertido o el papiloma velloso.
pesar de encontrarse l imitado al urotelio, por lo que es superficial, est
formado por clulas poco diferenciadas con displasia grave. Tiene una
alta tasa de recidiva y progresa hacia tumor i nfiltrante en el 50-75% de Diag nstico
los casos. Este mal pronstico le confiere un carcter completamente
distinto del carcinoma in siw de otras regiones, en las que se considera
el estadio inicial de la enfermedad tumoral. El carcinoma in situ puede la hematuria macroscpica o m icroscpica monosintomtica es el ha
estar asociado a focos de carcinoma superficial (26%) o nfiltrante (60%) llazgo ms frecuente, presente en el 75% de los pacientes (MIR 06-07,
o bien encontrarse de forma aislada, siendo generalmente multifocal 96). la presencia de microhematuria asintomtica, descubierta durante
tanto en vejiga como en otras puntos del uratelio (MIR 07-08, 1 02). estudios de cribado, slo se relaciona con enfermedad significativa en
menos del 2% de los casos. Pueden encontrarse sntomas irritativos
(escozor, polaquiuria, tenesmo) en el 25-30%, solos o acompaando a
la hematuria. la presencia de un sndrome cisttico no justificado por
i nfeccin o litiasis debe hacer sospechar la presencia de un carcinoma
Tis: carcinoma in situ (plano) vesical, especialmente por su asociacin con el carcinoma in situ (MIR
Ta: carcinoma papilar no infiltrante 00-01 F, 1 44). Con menor frecuencia, el paciente consulta por dolor en
T1: tumor que invade tejido conjuntivo subepitelial
T2: tumor que invade msculo
flanco por obstruccin ureteral, plvico o por edema en miembros in
- T2a: tumor que invade la mitad interna feriores (extensin l inftica). la exploracin fsica suele ser i rrelevante,
- T2b: tumor que invade la mitad externa salvo en la enfermedad avanzada.
T3: tumor que invade tejido perivesical:
T3a: microscpicamente

- T3b: macroscpicamente (masa extravesical) las citologas urinarias son una prueba sencilla y fiable que debe ser
T4a: tumor que invade prstata, tero o vagina rea l izada en todos los casos de hematuria asi ntomtica o sospecha de
T4b: tumor que invade pared plvica o pared abdominal
tumor vesical. Su sensibilidad depende del grado de diferenciacin
del tumor vesical, alcanzando el 75-1 00% en tumores de alto grado
y carcinoma in situ, siendo en este ltimo un mtodo diagnstico ms
Nx: metstasis ganglionar regional desconocida
NO: ausencia de metstasis ganglionar regional rentable que la ecografa, la Te, la urografa o la biopsia mltiple. Son
N1: metstasis a un solo ganglio entre 2 y 5 cm especialmente tiles en el seguimiento de pacientes sometidos a resec
N2: metstasis en un ganglio mayor de 5 cm o mltiples no mayores de 5 cm
cin transuretral en combinacin con la cistoscopia.
N3: metstasis mayores de 5 cm

Entre las pruebas radiolgicas destacan la ecografa (con una sensibi l i


MO: no metstasis dad del 80%, pera poco til para el d iagnstico de las neoplasias del
M l : metstasis a distancia tracto urinario superior (Figura 22]) y la urografa intravenosa (capaz de

Tabla 13. Estadificacin del carcinoma vesical


detectar la presencia de tumor en el 60% de los casos).

27
Manual eTO de Medicina y Ciruga, B.a edicin

un 40%, siendo tambin tratamiento de eleccin en el carcinoma


in situ, con una eficacia del 70% (MIR 05-06, 1 03; M I R 02-03, 1 79;
MIR 00-0 1 , 1 1 8). Sin embargo, no se utiliza en los tumores vesicales
como primera lnea, reservndose para los tumores de riesgo o con
mltiples recidivas, ya que tambin se acompaa de mayor nmero
de efectos secundarios y de complicaciones, entre los que se en
cuentran cistitis febril, sndrome pseudogripal, y las ms graves de
sepsis, prostatitis granulomatosa, neumonitis e incluso muerte. Estos
cuadros precisan tratamiento tuberculosttico completo al menos de
tres a seis meses.

En la enfermedad i nvasora O infiltrante, tras la reseccin transure


tral para evaluar la afectacin parietal, el tratamiento de eleccin
es la cistectoma radical. La quim ioterapia sistmica, adyuvante o
neoadyuvante, no parece mejorar los resultados de la ciruga aislada.
Igualmente se puede decir de la radioterapia, que nicamente ha de
mostrado i ncrementar el tiempo libre de recidiva local, s i n aumentar
la supervivencia.

La quimioterapia combinada con M-VAC (metotrexato, vinblastina,


adriamicina, cisplatino) se reserva para pacientes en los que existe
En la urografa i ntravenosa, adems de la presencia de defectos de afectacin ganglionar o metastsica, con respuestas completas en el
replecin, se puede sugerir el diagnstico de tumor vesical, rigidez 1 5-30% y parciales en el 30-40%. Tras la cistectoma, los urteres son
y falta de distensibilidad vesical, la obstruccin de un urter o el derivados generalmente a segmentos intestinales o a piel, pudiendo
desplazamiento de la vejiga, entre otros. En caso de dudas sobre el realizarse estomas no continentes o reservaras continentes directa
tracto urinario superior, se recurrir a la pielografa retrgrada en el mente a l remanente uretral.
momento de realizar la cistoscopia. sta es fundamental para la eva
luacin del tumor vesical; puede real izarse bajo anestesia local cuan
do existan dudas con las pruebas realizadas previamente, pero si el
o RECUERDA
la adriamicina (doxorrubicina) es un quimioterpico que puede produ
diagnstico de presuncin es firme, y dado que en todo tumor debe cir cardiotoxicidad.
realizarse reseccin transuretral para evaluar el grado de infi ltracin,
se puede esperar a tener al paciente en quirfano bajo anestesia gene
ral o raqudea para practicarla (MIR 00-01 F, 1 43). En un 1 0% de los
casos de cistoscopia con citologas positivas no se encuentra tumor
en la vejiga, lo cual puede ser debido a la presencia de carcinoma in 6.2. Tu mores del tracto u r i n a rio
situ, tumor en vas urinarias altas, a un carcinoma ductal de prstata
o a un falso positivo de la prueba (generalmente por inflamacin de s u perior
la pared vesical o por tratamiento concomitante con radioterapia o
quimioterapia endovesical).
Entre e l 2-5% d e los tumores uroteliales se encuentran localizados
La Te se utiliza para la estadificacin de la enfermedad infi ltrante, ya entre los cl i ces y los orificios ureterales. En su etiologa estn
que aporta informacin limitada sobre la infiltracin tumoral. La RM i m p l i cados los mismos factores que para el carcinoma vesical, a
consigue mejores imgenes de la cpula vesical por sus cortes sagita los que habra que aadir la nefropata por abuso de analgsicos
les, pero no aporta mayor informacin que la TC (fenacetinas) y la nefropata de los Balcanes. En el 70-80% de los
pacientes aparece hematuria macroscpica, siendo el dolor c
lico por obstruccin ureteral la segunda queja en frecuencia de
Tratamiento aparicin.

Todo el proceso diagnstico va encaminado a establecer si el tumor


o RECUERDA
la hematuria tambin es la manifestacin ms frecuente del hiperne
vesical es superficial o infiltrante, ya que el tratamiento vara radical froma.
mente en funcin de este hecho.

Los tumores superficiales son manejados mediante reseccin transu


retral . Dada la alta frecuencia de recidivas, la mayora se tratan poste Diag nstico
riormente con i nstilaciones endovesicales (quimioterapia o inmunote
rapia local) que disminuyan la aparicin de nuevos tumores. Entre los
quimioterpicos empleados se encuentran la mitomicina, la tiotepa, la La sistemtica d iagnstica es bsicamente la misma que para el
adriamicina o la epirrubicina. Con todos ellos se consigue reducir las tumor vesical. Es decir, ecografa (detectar hidronefrosis del lado
recidivas alrededor de un 20%. del tumor), UIV (se ver un defecto de replecin no compatible
con u n clculo o una anulacin funcional de ese sistema excretor),
La inmunoterapia endovesical con BCG (bacilo Calmette-Guerin) es, y Te para la estadificacin (Figura 23) ( M I R 98-99, 1 36; M I R 98-
sin duda, la ms eficaz, disminuyendo el porcentaje de recidivas en 99F, 1 45 ) .

28
UrOIOga
ureteropieloscopia, elemento que, adems de diagnstico, puede
tener un fin teraputico en tumores de pequeo tamao y aspecto
superficial (Figura 241.

Fumador + Hematuria

Sospecha d. tumor uroteUal

+
Citologias + Pruebas de imagen
<: ECO 'VP
UIV

t t
No concluyente Concluyente
para tumor vesical para tumor vesical

t t
Cistoscopia ES -------- ==. RTUV
'-=':= -;:
:: = :....' :::;--l
6t t t t
Biopsia vesical ES ----Tis
--. 2:12 TA,Tl

6t t


Sospechar tumor BCG Cistectomfa Quimio BCG (G3)
Tracto urinario superior
Revisiones
Ureterorrenoscopia
Cistoscopia
Pielografa retrgada ReCi iva TS ClOmia R diva _
+ Citologa
Citologas selectivas
Cepillado ureteral

Ftgura 24. Algoritmo diagostico-teraputko en tumor de vejiga (MIR 03-04,82)

Tratam iento

El tratamiento estndar es la nefroureterectoma radical con extirpacin


de un rodete perimetico vesical; tal extensin es necesaria por la ten
dencia a la recidiva de estos tumores. Es planteable, sin embargo, el
empleo de tratamientos conservadores en caso de tumores papilares,
Figura 23. Ecografia.Tumor vesical con zonas calcificadas no infiltrantes, nicos y de pequeo tamao.

En estos tumores, la citologa urinaria aumenta su eficacia si se ob Es necesario un seguimiento posterior de la vejiga y el rin contra la
tiene de forma selectiva, cateterizando el urter del lado afectado. teral, ya que un 20-30% desarrollarn un tumor vesical metacrnico y
Otros elementos de diagnstico son las biopsias por cepillado y l a un 2% en el sistema colector contralateral .

Casos clnicos representativos

Varn de 5 7 aos de edad, fumador, que consulta por hematuria terminal, polaquiu Mujer de 63 aos que es diagnosticada de carcinoma de clulas escamosas del Ir
ria, urgencia y dolor miccional. Presenta citologas urinarias positivas de carcinoma gono vesical, con invasin de la capa muscular. Cul seria su actitud leraputica
urotelial, y el estudio anatomopatolgico tras la reseccin transurelral es de carci en este caso?
noma in silu difuso, con intensa inflamacin crnica. El tratamiento estndar ser:
1) Radioterapia externa con 7.000 rads.
1 ) Instalacin del bacilo de Calmette-Guerin. 2) Quimioterapia adyuvante, seguida de cistectoma radical.
2) Cistectoma radical. 3) Reseccin transuretral, seguida de inmunolerapia intravesical (BCG).
3) Instilaciones con mitomicina. 4) Cistectoma radical con extirpacin de cara anterior de vagina.
4) Quimioterapia con cisplalino. 5) Radioterapia externa, seguida de quimioterapia con cisplatino.
5) Antiinllamatorios no esteroideos ms quinolonas durante seis meses.
MIR 03-04, 82; RC: 4
MIR 05-06, 1 03; RC: 1

29
Manual CTO de Medicina y Ciru g a, 8.... edicin

Casos clnicos representativos

Un paciente de 64 aos acude a consulta por sndrome miccional de dos meses de Una paciente de 58 aos acude por polaquiuria de dos aos de evolucin, nicturia
evolucin. Refiere que sU MAP le ha dado tratamiento antibitico y le ha realizado y dolor hipogstrico. Niega haber visto sangre en la orina y es fumadora. Seale la
cultivos que han sido negativos. Se le realiza una ecografa y una flujometra que son respuesta correcta:
normales y una urografa intravenosa que no evidencia alteraciones. En la cistosco
pia no se observan lesiones intravesicales. las citologas urinarias son sugestivas de 1) Se deben realizar biopsias a esta paciente.
malignidad. Seale la incorrecta 2) Una dstoscopia permitir confirmar el diagnstico.
3) los tratamientos empleados hasta el momento han demostrado una gran eficacia.
1) El tratamiento de eleccin ser la inmunomodulacin vesical. 4) Muy probablemente los cultivos sern positivos.
2) Es imprescindible la realizacin de biopsias para confirmacin del diagnstico. 5) En todos los casos se detectarn > 1 0 hemates/campo en el sedimento de orina.
3) Si recidiva tras las instilaciones, se deben repetir los ciclos dos veces ms.
4) la cistectoma es el tratamiento final en muchos de estos pacientes. Re: 1
5) A priori no se encontrar infiltracin de la capa muscular.

RC: 3

30
07.
TUMORES TESTICULARES

Ont>ntil(In Aspectos esenciales

MIR
Estc tema es sencillo m El tumor testicular es la neoplasia slida ms frecuente en el varn joven.
y rentable siempre
que se seleccione o la tasa de curacin es mayor del 90%.
lo realmente importante:
El tumor testicular ms frecuente es el seminoma. No obstante, sto es muy variable segn la edad del pa
la clnica y el diagnstico. [I)
Es fundamental la revisin ciente.
profunda de los Desgloses.
En cuanto al tratamiento, dado Una masa testicular por encima de los 50 aos debe hacer pens.lr en un linfoma.
que no existe un protocolo
universalmente aceptado, El tumor tpico de las disgenesias gonadales es el gonadoblasloma.
es mejor conocer ideas
generales. la clnica ms frecuente es una masa escrotal indolora.

El seminoma nunca produce afetoprotena .

Como tratamiento, la orquiectoma por va inguinal se realiza en todos los casos.

El seminoma es radiosensible. Por ello puede utilizarse radioterapia como tratamiento en los primeros esta
dios. Si se trala de un estadio avanzado, se empleara quimioterapia.
los lumores no seminomalosos se tratan con orquiecloma + quimioterapia. No obstante, si es un tumor
limitado al testculo, puede plantearse la vigilancia tras la orquiectoma.

7 . 1 . Eti ologa y epidemiolog a


Los tumores testiculares suponen el ' 2% de las neoplasias en varones, y son las neoplasias slidas ms frecuen
tes entre los 20 y 35 aos. Presentan mayor incidencia a mayor desarrollo de la sociedad, y tambin en la raza
blanca ms que en las dems. La tasa de curacin es superior al 90%.

El 95% de ellos proceden de clulas germinales y, aunque globalmente el semi noma es el ms frecuente, la in
cidencia vara segn el grupo de edad considerado. El 5% restante se reparte entre tumores del estroma gonadal
(1 2%), ! infamas (1 %), gonadoblastomas (clulas germinales y del estroma) metstasis y otros.

Los factores de riesgo para el desarrollo de tumor testicular son: teste h i poatrfico, sndrome de Klinefelter,
h i storia fam i l iar de neoplasias testiculares, tumor previo en el otro teste y criptorquidia. Los testculos no
descendidos tienen mayor riesgo de desarrollar tumores, y esta probabilidad aumenta si la situacin del
teste es intraabdominal. Asimismo, el testculo contra lateral, aunque de localizacin escrotal, tiene una
incidencia mayor; de hecho, el 20% de los tumores en pacientes con criptorquidia se desarrollan en el
testculo no criptorqudico. Por estas razones, los testculos no descendidos deben descenderse, preferi
blemente entre el primer y segundo ao de edad para facilitar su seguimiento y, con ello, la deteccin de
1 Preguntas un eventual tumor.

- MIR 09- 1 0. 102 Una vez alcanzada la pubertad, un teste criptorqudico no descendido probablemente deba extirparse, dado que
- MIR 06-07, 94
estos testculos pierden la capacidad de espermatognesis y conservan su potencial malignizante. No obstante,
- MIR 05-06. 107
- MIR 04-05, 107 hay autores que prefieren no extirparlos, siempre y cuando se puedan descender a la bolsa escrotal, ya que an
- MIR 03-04, 145 pueden mantener su funcin endocrina (secretora de testosterona) (MIR 9900, 1 86).
- MIR 02-03, 186
- MIR 01 -02, 108
- MIR 00-01 F, 142 Otros factores que se pueden encontrar relacionados son las hernias inguinales infantiles (no demostrado) y
- MIR 99-00, 186
- MIR 99-00F, 145
la orquitis urliana secundaria al paramixovirus causante de la parotiditis (siempre que haya producido atrofia)
- MIR 98-99F, 2 1 8 y, entre los factores txicos, la exposicin a radiaciones, fuentes de calor, productos para teido del cuero y

31
Manual eTO de Medicina y Ciruga, 8.3 edicin

estrgenos ntra tero durante el primer trimestre del embarazo (MIR saco vitelina en el 38% de los tumores testiculares del adulto.
98-99F, 2 1 8). Aunque entre un 8 y 25% de los pacientes presentan Coriocarcinorna. En el estudio histolgico tiene que contener ele
historia de traumatismo testicular, todos los autores parecen estar de mentos de sincitiotrofoblasto y citotrofoblasto para ser considerado
acuerdo en que ste supone ms bien el motivo por el que se descubre como tal. En el momento del diagnstico, generalmente existen me
una masa escrotal, y no su origen. tstasis a distancia (suelen ser va hematgena) y un tumor primario
testicular pequeo. Raramente es puro (MIR 06-07, 94).
Teratoma. Por definicin, se encuentra formado al menos por dos capas
distintas de clulas germinales (endodermo, mesodermo o ectodermo).
7.2. Anatoma patolgica Teratocarcinorna. Tumor mixto con reas de teratoma y de carcino
ma embrionario. Un 64% tiene tambin reas de semi noma.
Conadoblastoma. Contiene grandes clulas similares al seminoma
Los tumores testiculares se suelen originar como una masa testicular y otras menores, como clulas de Sertoli inmaduras o de la granulo
intraparenquimatosa. A partir de ah, pueden quedarse localizados o sa. Se asocia preferentemente a las gnadas disgenticas y estados
metastatizar. La localizacin ms frecuente de metstasis son los gan intersexuales.
glios retroperitoneales (casi siempre es el primer escaln en la disemi Tumores de clulas de Sertoli. Forman reas tubulares similares a
nacin) posteriormente puede aparecer la afeccin mediastnica, de los tbulos del testculo prepuberal normal. De comportamiento be
vsceras abdominales y de pulmn. nigno, es excepcional que metastaticen.

Hay que tener en cuenta que el testculo izquierdo drena a los ganglios
paraarticos y prearticos a nivel de L2. El derecho drena a ganglios
RECUERDA
El lumor de clulas de Serlol i se ha asociado al sndrome de Peutz
interaortocavos, precavas y prearticos, y tambin a nivel del hilio re Jeghers (vase Apa rtado de Sndromes de poliposis, en la Seccin de
nal. La diseminacin hematgena es menos frecuente, salvo en el co Digestivo y ciruga general).
riocarcinoma, va vasos espermticos, siendo los puntos ms habituales
de metstasis: pulmn, hgado, hueso y SNC, por orden de frecuencia.
Tumores de clulas de Leydig. Pueden verse cristaloides en su cito
Los tumores testiculares se dividen en dos grupos: tumores que no deri plasma. Generalmente son benignos. Pueden ser hormonal mente acti
van de las clulas germinales (5%) y tumores que derivan de las clulas vos, produciendo pubertad precoz o feminizacin (MIR 99-00F, 145).
germinales (95%); a su vez, estos ltimos se dividen en seminomatosos Linforna. Tanto como localizacin secundaria o como origen pri
y no seminomatosos (Tabla 1 4). mario (menos frecuente), una masa testicular en un varn mayor de
50 aos sugiere en primer lugar el diagnstico de linfoma. Tras la
TUMORES DE CHULAS GERMINALES TUMoRES DEL ESTRoMA orquiectoma o la biopsia testicular para el diagnstico de certeza,
Tumonos de un tipo
el tratamiento no vara respecto a los l infomas habituales.

$eminoma
- Tpico
- Anaplsico
- Espermatoctico 7.3. C l n i ca
Carcinoma embrionario
PoHembrioma
Tumor del co vitelina (seno endodrmico)
Coriocarcinoma Tumores clulas Leydig La manifestacin ms frecuente es como masa escrotal indolora. Con
Tumores clulas SertoH
Teratoma: mucha menor frecuencia, el motivo de consulta est originado por la

Tumores estructuras
- Maduro

gonadales primitivas presencia de metstasis ganglionares (masas supraclaviculares o abdo


- I nmaduro
- Con transformacin maligna minales (MIR 09-10, 1 02), o efectos endocrinos (ginecomastia, puber
tad precoz).
Tumores de m6s de un tipo hIstcJI6gIco

Teratocarcinoma
Otros
EI 1 0 % de los tumores se presentan como escroto agudo en la urgencia.

Tumores germinales + estroma'"

Gonadoblastoma RECUERDA
Exislen algunas enfermedades, como la sarcoidosis, que aumentan el
Tabla 14. Tumores de testkulo. Clasificacin histolgica
ta mao testicular sin existir un tumor.

Serninorna. Puede aumentar el tamao testicular hasta 1 0 veces sin


distorsionar su morfologa. Ocasionalmente se presenta extrates
tcular en mediastino (la ms frecuente), retroperitoneo, o regin
pineal de forma primaria. Se han descrito tres tipos histolgicos: 7.4. Diag nstico
seminoma tpico, anaplsico y espermatoctico (ms frecuente en
individuos mayores de 50 aos). El seminoma puro no es productor
de marcadores tumorales, pero hasta en un 1 5% de los casos pue La exploracin fsica de una masa indolora, sin signos inflamatorios y
den aparecer clulas del sincitiotrofoblasto, produciendo elevacio de largo periodo de evolucin, debe hacer sospechar un tumor testicu
nes de la p-HCG. lar (Tabla 1 5).
Carcinoma embrionario. Tiende a metastatizar de forma temprana.
Forma masas ms irregulares y heterogneas que otros tumores. Un elemento i mportante, tanto para su diagnstico como para el segui
Tumor del seno endodrmico. Este tumor fue descrito inicialmente miento, son los marcadores tumorales: a-fetoprotena (AFP) y fraccin
como una forma rara de tumor infantil. Se encuentran elementos de p de la gonadotropina corinica humana (P-HCG) (Tabla 1 6).

32
UrOIOga
presencia de enfermedad residual, ya que la elevacin persistente de
I ntratubular uno de estos marcadores despus del tratamiento supone la existencia
pTls
pTl Testculo y epididimo sin invasin vascular/linftica de tumor no eliminado.
pT2 Testculo y epiddimo con invasin vascular/linftica o tnica
vaginal La ecografa testicular es un mtodo sencillo y fiable para la diferen
pTl Afectacin de cordn espermtico
ciacin entre masas slidas y qusticas, y su localizacin exacta i ntra
pT4 Escroto
testicular o dependiente de los anejos. Cuando, a pesar de los marca
N1 Ganglios menores de 2 cm
dores, los datos ecogrficos son sugerentes de tumor, est indicada la
N2 Ganglios entre 2-5 cm
N3 Ganglios mayores de 5 cm exploracin quirrgica a travs de una incisin inguinal, para evitar la
posibi lidad terica de implantes tumorales en la piel escrotal y tener
M1. Metstasis en ganglios no regionales o pulmn
M1b Metstasis viscerales no pulmonares mejor control de pedculo vasculolinftico a nivel del cordn esperm
tico (MIR 01 -02, 1 08).
Sx Marcadores tumorales sricos no disponibles
SO Niveles de marcadores normales
51 lDH < 1.5 x n ; y p-HCG < 5.000; Y AFP < 1 .000 Si la exploracin confirma la presencia de una masa, el testculo debe
52 lDH entre 1,5 x n y 10 x n o p-HCG entre 5.000 y 50.000 ser extirpado (orquiectomfa radical).
o AFP entre 1 .000 Y 10.000
53 LDH > 10 x n o j3-HCG >50.000 o AFP > 1 0.000
La evaluacin de la extensin tumoral se completar mediante TC to
racoabdominal. As se constatar si la enfermedad est limitada al tes
tculo (estadio 1), o existe afectacin de ganglios infradiafragmticos
(estadio 11), o bien si hay incluso extensin supradiafragmtica o a r
ganos slidos (estadio 1 1 1). El sistema de estadificacin utiliza mltiples
variaciones, pero quiz la clasificacin ms aceptada sea la expuesta
anteriormente en la Tabla 1 5 .

7 . 5 . Diagnstico d ife rencial


Tabla 15. Estadificaci6n clnica y patolgica

Aunque frecuentemente l a existencia d e u n tumor testicular n o plan


Tumor con una palabra aumenta ji-HCG tea dudas diagnsticas, existen una serie de patologas testiculares
Coriocarcinoma que, junto con los tumores, pueden presentarse en algn momen
- Seminoma (en la mujer, el equivalente es el disgerminoma)
to dentro del cuadro genrico que se denomina "escroto agudo",
Tumor con dos palabras aumenta a.-fetoproteina caracterizado por el aumento doloroso de volumen del contenido
- Carcinoma embrionario escrotal, acompaado o no de signos inflamatorios (Tabla 1 7 y Fi
- Tumor del seno endodrmico
gura 25).
Tabla 16. Marcadores tumorales en tumores germinales Orquiepididimitis: suelen presentarse con dolor i ntenso, enro
(vlidos para ovario y testiculo) jecimiento cutneo, fiebre, y a veces si ntomatologa miccional.
En la exploracin, la elevacin del testculo (signo de Prehn)
La AFP es sintetizada por clulas del saco vitelino y, por tanto, est disminuye el dolor (signo de Prehn positivo). En su etiologa se
presente en tumores de saco vitelino o seno endodermal, y en los deben considerar grmenes de transmisin sexual en pacientes
carcinomas embrionarios. El seminoma nunca produce AFP (MIR adultos menores de 35 aos, y uropalgenos (E. coli) si superan
04-05, 1 07; M I R 02-03, 1 86). En cualquier caso, se debe tener en esla edad.
cuenta que la AFP es u n marcador i nespecfico, y se podra en Torsin del cordn espermtico: suele aparecer en la infancia o
contrar elevado en enfermedades hepticas benignas o malignas, la adolescencia, con dolor de aparicin brusca y signos cutneos
algunas neoplasias pancreticas y de la va biliar o en la ataxia inflamatorios crecientes a medida que progresa el cuadro. El teste
telangiectasia. se encuentra horizontal izado, y ocasionalmente puede palparse la
espiral del cordn torsionado. En este caso, la elevacin del testcu

a RlECUpERDAb ' . . '


lo i ncrementa la sensacin dolorosa.
Hidrocele y espermatocele: son dos cuadros que raramente se
a AF tam len se e Ieva en l IqUl-do amOlotlCO
.
en a IteraClones dei CIerre
del tubo neural. Ver Ginecologa. presentan de forma brusca y con dolor agudo, fcil mente diferen
ciables por la exploracin y su transiluminacin positiva y, ante la
duda, mediante ecografa.
La fraccin p de la HCG es producida por las clulas del sincitiotrofo
blasto presentes en el coriocarcinoma, y tambin de forma ocasional y
Polaridad conservada
de forma aislada, en algunos semi nomas (MIR 00-01 F, 1 42). Orquitis
Prehn MEJORA el dolor

En conjunto, el 70% de los tumores testiculares producen algn marca Testculo horizontalizado
Torsin del cordn espermtico
dor, luego existe hasta un 30% de tumores con marcadores negativos Prehn EMPEORA el dolor

al diagnstico.
Tumor testicular Masa palpable indolora
La vida media de la a-fetoprotena es de siete das, frente a tres das
Tabla 17. Diagnstico diferencial de los tumores testiculares
de la -HCG. Este dato es importante a la hora de valorar la posible

33
Manual eTO de Medicina y Ciruga, 8.a edicin

D RECUERDA
El cisplatino produce vmitos con mucha frecuencia. Otro efecto secun
dario es su nefrotoxicidad.

Estadio lIa-lIb. En este caso, el tumor ya est extendido a gangl ios


retroperitoneales, y requiere, por tanto, tratamiento agresivo. Se
dispone de dos opciones: radioterapia sobre las cadenas afectas
(teniendo en cuenta el teste afectado, se irradiar a unas cadenas
ganglionares u otras), o quimioterapia BEP (cisplatino, etopsido y
bleomicina). Ambos obtienen resultados muy similares
Estadio 11e-1II (estadios avanzados). El tumor tiene metstasis gan
glionares retroperitoneales superiores a 5 cm, O afectacin supra
diafragmtica o de vsceras slidas. La radioterapia deja de ser una
opcin teraputica. La quimioterapia (BEP) es la nica posibilidad,
presentando una tasa de curacin de alrededor del 80%.

Tu mores no seminomatosos

Estadio 1. Existen tres posibilidades teraputicas tras la orquiectoma:


Observacin y seguimiento peridico. Tasas de recadas de un 20%.
linfadenectoma retroperitoneal de estadificacin, pues as se
tiene certeza del estadio y se reduce a un 1 0% las recidivas (en
Europa no est extendida esta prctica).
Quimioterapia profilctica (cisplatino), y as se reducen al 5%
Figura 25. Pieza de tumor testicula r las recidivas.
En aquellos casos en que exista invasin vascular en la pieza de
orquiectoma, la tasa de recidivas asciende al 50%. Parece lgico,
en estos casos, inclinarse de entrada por una de las dos ltimas op
7.6. Trata m i e nto ciones. La tasa de curacin alcanza el 98%.
Estadio lIa-lIb. Histricamente se realizaba linfadenectoma retro
peritoneal completa como nico tratamiento, pero ante tasas de
Hasta la fecha no existe un protocolo nico de tratamiento, pudiendo recidiva no desdeables, actual mente se indica quimioterapia de
variar incluso de un centro a otro. Siempre se realizar orquiectoma inicio (BEP). La tasa de supervivencia supera el 95%.
radical va inguinal. Posteriormente, el patlogo informar de la estir Estadio 11e-1II. Antes de la aparicin de la actual quimioterapia, la
pe histopatolgica. A continuacin, el estudio de extensin mediante supervivencia era del 51 0%. Actualmente, el esquema de trata
Te toracoabdominoplvica y nuevos marcadores postorqu iectoma. En miento mayoritariamente aceptado es quimioterapia primaria.
funcin de la histologa y del estadio, se asignar un tratamiento a cada
enfermo. El tratamiento de la neoplasia testicular se resume en la Figura 26.

D RECUERDA TUMOR TESTICULAR


La orquiectoma en el cncer de testculo es v a inguinal. En el cncer
de prstata , va escrotal.
t
Orquiectoma radical

NO SEMINOMA
Seminoma SEMINOMA

t t t t
la tla,lIb
lIa,lIb

Se caracteriza por su gran radiosensibilidad, de ah que la radioterapia Observacin


t Observacin +++ t
Si FR : Linfadenectomra invasin Quimioterapia
haya sido la base del tratam iento de estos tumores. Actualmente, l a
Radioterapia
> 4 cm rere testis Quimioterapia
(EEUUI vascular
quimioterapia obtiene resultados similares. Quimioterapia o
Estadio l. El tumor tericamente est l imitado al testculo. No obs (UEI linftica
Radioterapia
tante, se sabe que hasta un 20% de pacientes presentan micro
Quimioterapia
metstasis a ganglios retroperitoneales, que en el momento del
diagnstico no se detectan. La presencia de micrometstasis se ha SEMINOMA I NO SElIllfIOMA
relacionado con dos factores de riesgo: tumor de ms de 4 cm y la
invasin tumoral de la rete lestis. En estos enfermos se aconseja ad
ministrar radioterapia o quimioterapia (cisplatino). Con sto, frente Quimioterapia
a los pacientes que optaron nicamente por observacin, la recidiva
desciende del 20 al 5%. Figura 26. Algoritmo de tratamiento de la neoplasia testicular

34
urOlogja
Masas residuales

Se define como masa residual la existencia de conglomerados adeno MASA RESIDUAL

pticos tras tratamiento quimioterpico o radioterpico. Cuando el tu


mor primario es un semi noma y existen masas residuales, la actuacin
a segui r ser: si la masa residual es inferior a 3 cm, tiene muy pocas SEMINOMA NO SEMINOMA

probabi lidades de contener tumor residual y no requiere ms que ob


servacin. Pero cuando es superior a 3 cm, se debe realizar una PET < 3 cm > 3 cm
(tomografa por emisin de positrones), si existiera esta posibilidad a
+
nivel tcnico, pues detecta con una alta sensibilidad y especificidad la
presencia de tumor residual. Si no se dispone de una PET o si sta es
positiva, se realizar ciruga de la masa.
1
Observacin -+---
+/- PET

e ;/ "'- $

Cuando el tumor primario es un tumor no seminomatoso, se debe rea



Exresis de la masa residual
lizar exresis de dicha masa siempre, con independencia del tamao.

La histologa de estas masas residuales, una vez extirpadas, puede ser: 50% necrosis 35% teratoma 15% tumor viable

tejido necrtico hasta en un 50% de las ocasiones, tumor viable en un


1 5% y teratoma en un 35% (stos, dejados a libre evolucin, pueden
convertirse en teratomas malignos o producir procesos compresivos
Figura 27. Algoritmo de tratamiento de las masas residuales
con su crecimiento) (Figura 27).

Casos clnicos representativos

Un hombre de 31 aos de edad consulta por la presencia de una masa palpable en Qu diagnstico, entre los siguientes, es el ms probable en un nio de 8 aos con
el teste derecho, de un mes de evolucin, no dolorosa. Su urlogo le realiza una signos inequvocos de pubertad precoz y que, en la exploracin, presenta una masa
ecografa testicular, en la que se evidencia una lesin hipoe<oica, bien delimitada, en cl testculo derecho de 2 cm de diilmetrol
intratesticular. los marcadores tumorales a-fetoproteina y HCG son negativos. la
actitud ms correcta de, entre las siguientes, sera: 1 ) Tumor de clulas de leydig.
2) Scminoma.
1) Dado que los marcadores tumorales son negativos, se descarta neoplasia testicular 3) Tumor del saco vitelina.
y requiere observacin. 4) Teraloma.
2) Repetir la ecografa testicular en un plazo de tres meses. 5) Coriocarcinoma.
3) Realizacin de una tomografa axial compularizada loraco-abdrnino-plvica.
4) Biopsia transeseralal dcl tcstculo. MIR 03-04, 1 45; Re 4
5) Orquiectoma radical y esperar resultado del patlogo.

MIR 05-06. 107; Re: 5

35
08.
TRAS P LANTE RENAL

Orlentaclon

MIR
lo ms imJX)rtante de
este tema son los tipos de OJ las causas ms frecuentes de insuficiencia renal crnica son la diabetes mellitus y las glomerulonefritis.
rechazo, que se solap...n con
Inmunologla. Se debe insislir
en el rechazo agudo, que o la clnica caracterstica del rechazo agudo es: fiebre, hipertensin y dolor en el rea del injerto.
conviene repasar con las
preguntas de aos anleriOfe5. GJ El rechazo agudo produce oliguria, no poliuria.

8 . 1 . I nd i caciones

las dos enfermedades que ms comnmente abocan a una insuficiencia renal terminal irreversible, tratable
mediante un trasplante renal, son la glomerulonefritis y la diabetes mellitus insuli nodependiente.

Otras causas importantes son:


Poliquistosis renal.
Nefroesclerosis hipertensiva.
Enfermedad de Alport.
Nefropata IgA.
lupus eritematoso sistmico.
Nefroesclerosis.
Nefritis intersticial.
Pielonefritis.
Uropata obstructiva.

los mejores receptores son individuos jvenes cuyo fallo renal no se deba a una enfermedad sistmica que pue
da daar el rin trasplantado o causar la muerte por causas extrarrenales. Generalmente se suele mantener al
receptor en tratamiento con dilisis durante un cierto tiempo previo al trasplante.

8.2. Contra i n d icaciones

las contraindicaciones absolutas son las siguientes:


Infeccin activa.
Enfermedad maligna que no pueda ser erradicada.
Sospecha de no cumplimiento teraputico del protocolo inmunosupresor.
Glomerulonefritis activa.
Expectativa de vida reducida por enfermedad de base del paciente.
Presencia de anticuerpos preformados frente a antgenos del donante.

7 Preguntas
En referencia a las contraindicaciones relativas del trasplante renal se debe decir que stas se han ido modifi
- MIR 02-03, 178
cando a lo largo de los aos, al mejorar la tcnica y los cuidados prequirrgicos y postquirrgicos. En muchas
- MIR 99-00F, 1 4 1 ocasiones, el trasplante plantea menos riesgo que una hemodilisis crnica.

36
urOloga
Actualmente se consideran contraindicaciones relativas la edad avan Complicaciones tcnicas. Complicaciones vasculares, hemorragia,
zada, la oxalosis, la amiloidosis, la enfermedad iliofemoral oclusiva, hipertensin por estenosis de la arteria renal, trombosis venosa,
las anomalas del tracto urinario inferior O las alteraciones psiquitricas complicaciones del tracto urinario, necrosis tubular aguda, linfo
graves (MIR 99-00F, 1 4 1 ). celes.
Complicaciones no tcnicas. Infecciones bacterianas y oportunis
tas en relacin con la inmunosupresin, hiperglucemias, compli
caciones gastrointestinales, h i perparatiroidismo y tumores (cncer
8.3. Com p l i caciones de piel y de labios, carcinoma in silU de crvix, l infomas no
Hodgkin; guardan relacin con el tratamiento inmunosupresor).
Puede aparecer h ipertensin debida a enfermedad en los riones
Las complicaciones que se pueden presentar son las siguientes: originales, como consecuencia de rechazo, por estenosis de l a
Rechazo (Tabla 1 8) (MIR 02-03, 1 78). anastomosis d e l a arteria renal o por toxicidad renal por ciclos
Recurrencia de la enfermedad en el rin trasplantado. porina.

RECHAZO INICIO PATOGENIA PA TRATAMIENTO

Minutos. das Ac. preformados Trombosis microvascular Nefrectoma del injerto


CID Isquemia o infarto
HIpoIagudD Act. del complemento PMN en capilares
Dao endoletelial

Das Celular (+Ac) Vasculitis necrotizante Bolos de esteroides


-- Respuesta 2. a Ag-HLA Ac monoclonales

Semanas Celular (+Ac) Forma vascular: mediada por Ac Bolos de esteroides (la vascular suele ser resistente)
Agudo Infiltrado de linfocitos Forma celular: tubulointersticial Ac monoclonales

Meses. aos Humoral y celular Intima arterial aumentada No hay; control de HTA
CnInk:o Atrofia tubular
Glomerulopata

Tabla 18. Rechazo en el trasplante renal

Un paciente de 35 aos, con insuficiencia renal crnica, secundaria a pielonefritis 1) Crisis hipertensiva.
crnica recibe un trasplante renal de cadver con el que comparla dos idenlidades 2) Infeccin respiratoria.
en A y 8 Y una en DR. Recibe tratamiento inmunosupresor con cidosporina A y cor 3) Pielonefritis aguda del injerto renal.
ticoides en dosis estndar. En el posoperatorio inmediato se observa buena diuresis, y 4) Recidiva de su enfermedad renal.
no es necesario el tratamiento sustitutivo con hemodilisis. En el 5." da de evolucin, 5) Rechazo agudo del injerto renal.
el paciente pt"esenta fiebre de 38", lA de 180/110, oliguria y disminucin en la con
centracin urinaria de sodio. El diagnstico ms probable sera: MJR 02-03, 1 78; RC: 5

37
09.
UROPATA OBSTRUCTIVA

Oflentaclon

MIR
Tema poco preguntado hasta
la fecha. Se debe tener una
GJ la uropata obstructiva puede producir insuficiencia renal si no se resuelve a tiempo.

idea general y aprender los


Aspectos esenciales. (TI El dolor suele estar presente en la obstruccin aguda. Sin embargo, en la crnica, es frecuente su ausencia.

(l) Despus de resolver una uropata obstructiva puede producirse una fase de poliuria.

9 . 1 . Caractersticas

Detencin del flujo d e orina e n cualquier punto entre los clices renales y e l exterior (Tabla 1 9).
Su i mportancia reside en el desarrollo potencial de insuficiencia renal, por lo que tiene importancia la obs
truccin urinaria bilateral o la unilate-

RECUERDA
ral sobre rin nico funcionante.
Una obstruccin de ms de un mes
la uropata obstructiva puede producir glomerulonefrltis focal y
de duracin puede dar lugar a u n segmentarla.
dao renal funcional y estructural
permanente.

EXTRAPARIETAL EXTRAPARIETAl
INTRAlUMINAl INTRAPARIETAl
(compreslon extnnseca) (dlsfunclon neuromuscular)

litiasis Estenosis congnita Urter retrocavo Vejiga neur6gena


Tumores (hipernefroma, Estenosis postinfecciosa RiMn en herradura Vejiga automtica: lesin sobre
uroteliomal Estenosis postraumtica Fibrosis retroperitoneal L1-L3
Necrosis papilar Estenosis isqumica . Tumores (prstata, Vejiga tona: lesin bajo ll-L3
Cogulos vejiga, ginecolgicos) Disfuncin de la unin
Hiperplasia prosttica pieloureteral
ligadura iatr6gena Reflujo vesicoureteral
de urteres

Tabla 19. Clasificacin de la uropatfa obstructiva

9.2. C l n ica (MIR 98-99F, 1 38)

La forma de presentacin depende de los siguientes factores:


Etiologa de la obstruccin. Presenta la clnica propia de la enfermedad de base.
Tiempo de evolucin. La aguda suele cursar con dolor (clico nefrtico), siendo la crnica ms frecuentemen
te asintomtica.
Lugar de obstruccin:
Tracto urinario inferior (uretra y vejiga). Cursa con retraso para iniciar la miccin, disminucin de fuerza
y del tamao del chorro, goteo terminal, hematuria, escozor al orinar, orina turbia, retencin aguda de
, Preguntd<;
orina o i ncontinencia paradj ica ("miccin por rebosamiento").
MIR 07..Q8, lOS
Tracto urinario superior (urter y rin). Estos pacientes presentan dolor en el flanco (rin y urter proxi
. MIR 98-99f, 138 mal), dolor en flanco con irradiacin a genitales (urter medio) o sndrome miccional (urter terminal) .

38
urologia
Despus de resolverse una obstruccin, sobre todo si es crnica, puede y su reversibil idad. En los casos en que hay destruccin irreversible de
producirse una fase de poliuria. Esto se debe a que, a nivel tubular, la va urinaria, es necesario realizar una derivacin urinaria definitiva
cuando se ralentiza crnicamente el flujo urinario, se genera una in (Figura 28).
sensibilidad a la ADH transitoria (diabetes inspida nefrognica), de ah
la poliuria.
MANEJO DE LA UROPATIA OBSTRUCTIVA

.
9.3. D i a g nstico INFRAVESICAl SUPRAVESICAl
(globo vesical) (no globo vesical)

El diagnstico d e seguridad, la valoracin d e la evolucin, y el pro


+ t
SONDA VESICAL
nstico son ecogrficos. Adems, son tiles la anamnesis y la explo
O ClSTOSTOMIA
racin fsica, la radiologa simple (conveniente en litiasis radioopaca),
urografa i ntravenosa (confirma una posible anomala funcional y til
en litiasis radiotransparente), cistografa, estudio metablico (til en

A
No dilatacin Dilatacin unilateral Dilatacin
prevencin de recidivas), Te abdominal, ecografa transrectal, biopsia bilateral

j
prosttica dirigida, pielografa retrgrada, nefrostografa, cistoscopia,
Estudio mdico
flujometra, cistomanometra y citologa urinaria (MIR 07-08, 105).
Birreno Monorreno

Estudio (UIV, TC)


9.4. Trata m iento Fracaso renal

Derivacin de va
Es necesario restablecer el flujo urinario. La mayor parte de las veces urinaria superior
(nefrostomfa/doble J)
se realiza mediante litotricia o correccin quirrgica. Si la obstruccin
es aguda y/o bilateral, la desobstruccin es urgente, y puede lograrse
mediante un sondaje vesical, talla vesical, catter ureteral o nefrosto
Figura 28. Procedim;ento de actuacin frente a la uropatla obstructiva
ma. En caso contrario, hay que valorar el grado de sufrimiento renal

39
1 0.
DISF UNCiN ERCTIL

Orl(>ntaclon

MIR
Tema de reciente
introduccin, sencillo
(jJ la causa ms frecuente de disfuncin erctil es vascular.
y muy rentable. Se debe
incidir en los factores de o la enfermedad endocrina ms relacionada con ella es la diabetes mellitus.
riesgo, el tratamiento El sildenafilo est contraindicado en pacientes que toman nitratos o frmacos donadores de xido ntrico, en
y sus contraindicaciones. lIJ
pacientes con infarto agudo de miocardio (lAM) en los ltimos seis meses, y en pacientes con insuficiencia
Es conveniente conocer muy
bien las preguntas aparecidas cardaca grave o angina inestable.
con anterioridad.

1 0. 1 . I ntrod uccin

La disfuncin erctil (DE) se define como l a incapacidad persistente o recurrente para conseguir o mantener l a sufi
ciente rigidez del pene que permita una relacin sexual satisfactoria. Debe tener una duracin mnima de tres meses.

1 0.2. Preva lencia

En Estados Unidos, en u n estudio en varones de entre 40 y 70 aos, se estim que l a prevalencia global era del
52%. La prevalencia en Espaa se estima en 1 .500.000 a 2.000.000 varones, en torno al 1 2, 1 %.

1 0.3. Etiologa

Se puede clasificar en (MIR 04-05, 1 08):


Orgnica. Causas vascu lares (las ms frecuentes 60-80%), neurolgicas ( 1 0-20/o), hormonales (5-1 0%) o
locales.
Psicgena.
Mixta. En la mayora de los casos de etiologa orgnica se aade un componente psicolgico.

1 0.4. Factores de riesgo

Edad: factor independiente.


Diabetes: es la enfermedad endocrina ms frecuente asociada a disfuncin erctil. Significa una probabilidad tres
veces superior de presentar DE. Estn implicados mecanismos vasculares, neuropticos y disfuncin gonadal.
Enfermedad cardiovascular: cardiopata, hipertensin arterial, enfermedad vascular perifrica y descenso del
7 Preguntas colesterol HDL se han relacionado de manera clara con la d isfuncin erctil.
Tabaquismo: factor independiente.
- MIR 06-07, 97
- MIR 05-06, 108 Secundaria a frmacos: aquellos que causan hiperprolactinemia, que disminuyen los niveles de testosterona,
- MIR 04-05, 108 psicotropos y antihipertensivos.

40
urologia a

Secundaria a consumo de drogas: cocana, herona, etc. un inhibidor de la fosfodiesterasa tipo 5 (PDE5). Induce la relaja
Trastornos afectivos: depresin. cin del msculo liso del cuerpo cavernoso, l iberando xido ntrico
(NO). El NO liberado por el endotelio vascular y por las terminacio
nes nerviosas no adrenrgicaslno colinrgicas es el principal neu
rotransmisor de la ereccin. Precisa de deseo sexual y estimulacin
1 0. 5 . Diag nstico previa para su efecto.

Las contraindicaciones absolutas de sildenafilo son:


El diagnstico debe basarse en los siguientes componentes: Administracin concomitante con nitratos o frmacos donadores
Historia clnica y sexual: investigar los posibles factores de riesgo de xido ntrico por el riesgo de hipotensin grave (dinitrato/mo
implicados. nonitrato de isosorbide, molsidomina, nicorandil, nitroglicerina,
Exploracin fsica: encaminada a descartar enfermedad vascu nitroprusiato sdico) (MIR 06-07, 97; MIR 05-06, 108).
lar, enfermedades neurolgicas, trastornos genitales y endocri Pacientes en los que est desaconsejada la actividad sexual (an
nopatas. En varones mayores de 50 aos se incluir tacto rectal. gina inestable, insuficiencia cardaca o i nfarto reciente, hace
Determinaciones analticas: glucemia basal, perfil l ipdico, testos menos de seis meses).
terona total y l ibre y prolactina. Adems, es conveniente solicitar
hemograma, funcin renal y heptica. Apomorfina: agonista dopaminrgico que acta a nivel central so
Pruebas especializadas: nicamente en ocasiones muy seleccionadas. bre el mecanismo de la ereccin. Est contraindicado en sujetos que
tengan desaconsejada la actividad sexual.

1 0.6. Trata m i e nto Seg unda lnea

Se puede estructurar en tres escalones o etapas, que sern superadas en Terapia intracavernosa: alprostadil (PGE1), mediante inyeccin di
funcin de fracaso del escaln previo. recta en 105 cuerpos cavernosos. Otros frmacos son la papaverina
y la fentolamina.

Frmacos orales
Tercera lnea

Citrato de si ldenafilo: se considera actualmente como el tratamien


to farmacolgico de eleccin en la DE. Hoy en da existen nuevos Ciruga de revascularizacin (venosa, arterial). Implante de prtesis
frmacos basados en l, como vardenafilo y tadalafilo. Se trata de de pene.

Paciente de 6J aos, en tratamiento a demanda con citrato de sildenafilo por presen J) Digoxina.
tar disfuncin erctil de aos de evolucin. Seale cul de los siguientes frmacos 4) lndapamida.
NO asociara en ningn caso a su tratamiento: 5) Mononitrato de isosorbide.

1 ) Amiodarona. MJR 05-06, 108; RC: 5


2) Verapamilo.

41

También podría gustarte